Anatomy Exam 1 (Lecture)

Lakukan tugas rumah & ujian kamu dengan baik sekarang menggunakan Quizwiz!

B. around the middle of the second trimester

"Quickening" or the period when fetal movements are commonly first felt by the mother occurs: A. near the end of the first trimester B. around the middle of the second trimester C. at the end of the second trimester D. at the end of the embryonic period E. at the end of the fetal period

C. Internal Thoracic Artery

35 year old male presents with a history of thoracic cage trauma after a car accident. The thoracic surgeon needs to repair an active bleeding secondary to rupture of the vascular structure labeled as X A. Anterior Intercostal Artery B. Posterior Intercostal Artery C. Internal Thoracic Artery D. Perforating Cutaneous Artery

C. Posterior intercostal membrane

45 year old woman with metastatic lung cancer presents with severe respiratory distress secondary to massive right side malignant pleural effusion. You decide to insert a chest tube to drain the pleural cavity. During the procedure, the needle pierces several structures. Which of the following structures is less probable that you pierced at the midaxillary line. A. Internal intercostal muscle B. Innermost intercostal muscle C. Posterior intercostal membrane D. External intercostal muscle

E. Stenosis of the aorta Stenosis of the aorta can cause left ventricular hypertrophy. Right ventricular hypertrophy may occur as a result of pulmonary stenosis, pulmonary and tricuspid valve defects, or mitral valve stenosis.

A 12-year-old boy was admitted to a local hospital with complications from his congenital heart disease, including left ventricular hypertrophy, which could have resulted from which of the following conditions? A. A constricted pulmonary trunk B. An abnormally small left AV opening C. Improper closing of the pulmonary valves D. An abnormally large right AV opening E. Stenosis of the aorta

C. Inferior pancreaticoduodenal artery The inferior pancreaticoduodenal artery is a branch of the superior mesenteric artery. All of the other arteries are branches of the splenic artery.

A 16-year-old boy with a ruptured spleen comes to the emergency department for splenectomy. Soon after ligation of the splenic artery just distal to its origin, a surgical resident observes that the patient is healing normally. Normal blood flow would occur in which of the following artery? A. Short gastric arteries B. Dorsal pancreatic artery C. Inferior pancreaticoduodenal artery D. Left gastroepiploic artery E. Artery in the lienorenal ligament

E. Kidney The urinary system includes the kidneys, which remove wastes from the blood and produce the urine; the ureters, which carry urine; the urinary bladder, which stores urine; and the urethra, which conveys urine from the bladder to the exterior of the body. The spleen filters blood to remove particulate matter and cellular residue, stores red blood cells, and produces lymphocytes. Because the patient is not producing urine properly, the malfunctioning organs are the kidneys.

A 16-year-old girl comes to a local hospital with urinary urgency and frequency. Her urologist's examination and laboratory test results reveal that she has difficulty in removing wastes from the blood and in producing urine. Which of the following organs may have abnormal functions? A. Ureter B. Spleen C. Urethra D. Bladder E. Kidney

A. T10 on the left

A 17 year old boy is brought to the emergency department from a high school football game. The examination reveals a loss of vibratory sensation and discriminative touch on the left lower extremity and to the level of the umbilicus. CT shows a vertebral fracture with bone displacement into the vertebral canal. Which of the following indicates the most lkely level of damage to the spinal cord in this boy? A. T10 on the left B. T8-T9 on the right C. T12 on the left D. T10 on the right E. T7-T8 on the left

D. Dorsal root ganglion and lateral horn of the spinal cord The white rami communicantes contain preganglionic sympathetic VE fibers and VA fibers, whose cell bodies are located in the lateral horn of the spinal cord and the dorsal root ganglia. The sympathetic chain ganglion contains cell bodies of the postganglionic sympathetic nerve fibers. The anterior horn of the spinal cord contains cell bodies of the SE fibers. The dorsal root ganglion contains cell bodies of both SA and VA fibers.

A 17-year-old boy was involved in a gang fight, and a stab wound severed the white rami communicantes at the level of his sixth thoracic vertebra. This injury would result in degeneration of nerve cell bodies in which of the following structures? A. Dorsal root ganglion and anterior horn of the spinal cord B. Sympathetic chain ganglion and dorsal root ganglion C. Sympathetic chain ganglion and posterior horn of the spinal cord D. Dorsal root ganglion and lateral horn of the spinal cord E. Anterior and lateral horns of the spinal cord

Skin --> superficial fascia --> deep fascia --> anterior rectus sheath --> rectus abdominis muscle --> transversalis fascia --> extraperitoneal connective tissue and peritoneum

A 18 year old primigravid woman in her 37th week of pregnancy arrived in the emergency room following an accident and she went into labor. The accident had broken her pelvis in such a way that the ER physician deemed a vaginal delivery would be hazardous. An obstetrician was called, and she agreed with ER physicians initial assessment. A C section was performed, resulting in the delivery of a healthy baby girl. During the operation, the obstetrician used a Pfannenstiel incision to open the abdomen. The incision involves making a transverse, slightly convex cut large enough to deliver a child at approximately the pubic hairline What abdominal wall layers must be incised at the pubic hairline (near the midline) in order to access the abdominal cavity?

D. All of the above

A 18 year old primigravid woman in her 37th week of pregnancy arrived in the emergency room following an accident and she went into labor. The accident had broken her pelvis in such a way that the ER physician deemed a vaginal delivery would be hazardous. An obstetrician was called, and she agreed with ER physicians initial assessment. A C section was performed, resulting in the delivery of a healthy baby girl. During the operation, the obstetrician used a Pfannenstiel incision to open the abdomen. The incision involves making a transverse, slightly convex cut large enough to deliver a child at approximately the pubic hairline Why is the incision made in a convex manner instead of straight across? A. Avoid the inguinal ligament and its contents B. To minimize the number of nerves that are damaged reducing post-operative muscle paralysis and skin paresthesia C. Segmental nerves of the anterior abdominal wall follow an inferior to superior course as they progress laterally D. All of the above

C. Optic nerve Multiple sclerosis affects only axons in the CNS (spinal cord and brain) that have myelin sheaths formed by oligodendrocytes. The optic nerve is considered to be part of the CNS because it is derived from an outpouching of the diencephalon. All other nervous structures are in the PNS and have their myelin sheaths formed by Schwan cells.

A 19-year-old man is diagnosed with multiple sclerosis (MS). Which of the following nervous structures would most likely be affected by this disease? A. Trigeminal ganglion B. Superior cervical ganglion C. Optic nerve D. Facial nerve E. Spinal accessory nerve

B. Middle cardiac vein The middle cardiac vein ascends in the posterior interventricular groove, accompanied by the posterior interventricular branch of the right coronary artery. The great cardiac vein is accompanied by the anterior interventricular artery; the anterior cardiac vein drains directly into the right atrium; and the small cardiac vein is accompanied by the marginal artery.

A 19-year-old man was brought to the emergency department after a motor vehicle accident, and his angiogram exhibited that he was bleeding from the vein that is accompanied by the posterior interventricular artery. Which of the following veins is most likely to be ruptured? A. Great cardiac vein B. Middle cardiac vein C. Anterior cardiac vein D. Small cardiac vein E. Oblique veins of the left atrium

C. Ileocolic artery The appendicular artery is a branch of the ileocolic artery. The other arteries do not supply the appendix. The middle colic and right colic arteries are branches of the superior mesenteric artery. The inferior mesenteric artery passes to the left behind the peritoneum and distributes to the descending and sigmoid colons and the upper portion of the rectum. The common iliac arteries are bifurcations from the aorta.

A 19-year-old man with a ruptured appendix is sent to the emergency department for surgery. To cut off the blood supply to the appendix (if collateral circulation is discounted), a surgeon should ligate which of the following arteries? A. Middle colic artery B. Right colic artery C. Ileocolic artery D. Inferior mesenteric artery E. Common iliac artery

D. Left colic flexure The vagus nerve supplies parasympathetic nerve fibers to the GI tract and terminates approximately at the left colic (splenic) flexure (junction of the transverse colon and the descending colon). The duodenojejunal junction, ileocecal junction, and right colic flexure are supplied by the vagus nerve. The descending colon, sigmoid colon, rectum, anal canal, and anorectal junction are supplied by the pelvic splanchnic nerve for parasympathetic innervation.

A 19-year-old woman with a long history of irritable bowel syndrome presents for the possibility of surgical resection of the gastrointestinal (GI) tract where the vagal parasympathetic innervation terminates. Which of the following sites is most appropriate for surgical resection? A. Duodenojejunal junction B. Ileocecal junction C. Right colic flexure D. Left colic flexure E. Anorectal junction

B. Deep inguinal ring The deep inguinal ring lies in the transversalis fascia, just lateral to the inferior epigastric vessels. The superficial inguinal ring is in the aponeurosis of the external oblique muscle. The inguinal ligament and the anterior wall of the inguinal canal are formed by the aponeurosis of the external oblique muscle. The sac of a direct inguinal hernia is formed by the peritoneum.

A 2-year-old boy presents with pain in his groin that has been increasing in nature over the past few weeks. He is found to have a degenerative malformation of the transversalis fascia during development. Which of the following structures on the anterior abdominal wall is likely defective? A. Superficial inguinal ring B. Deep inguinal ring C. Inguinal ligament D. Sac of a direct inguinal hernia E. Anterior wall of the inguinal canal

B. T11, subcostal, iliohypogastric and ilioinguinal nerves

A 21 year old female student presents to the emergency room with a complaint of "colicky" periumbilical pain which has intensified over the last 6-8 hours and now has started to migrate to the right lower quadrant. The patient reports some initial nausea, and as the pain has increased she has had increasing emesis and anorexia Physical exam demonstrates the patient has no distension, auscultation reveals hyperactive bowel sounds, and on palpation the patient demonstrates abdominal guarding and rebound tenderness, and the muscles of the anterior wall in the right lower quadrant are rigid In addition, the patient has low grade fever, and laboratory tests reveal a rising white blood cell count. The attending determines that the patient has acute appendicitis and prepares to take the student to the O.R. for an appendectomy. The surgeon asks you the following questions regarding the surgery. During an incision in the abdominal wall, what nerves have to be identified? A. intercostal, ilioinguinal and iliohypogastric nerves B. T11, subcostal, iliohypogastric and ilioinguinal nerves C. T12, subcostal, iliohypogastric and ilioinguinal nerves D. T10, T11, iliohypogastric and ilioinguinal nerves

E. iliohypogastric and ilioinguinal nerve

A 21 year old female student presents to the emergency room with a complaint of "colicky" periumbilical pain which has intensified over the last 6-8 hours and now has started to migrate to the right lower quadrant. The patient reports some initial nausea, and as the pain has increased she has had increasing emesis and anorexia Physical exam demonstrates the patient has no distension, auscultation reveals hyperactive bowel sounds, and on palpation the patient demonstrates abdominal guarding and rebound tenderness, and the muscles of the anterior wall in the right lower quadrant are rigid In addition, the patient has low grade fever, and laboratory tests reveal a rising white blood cell count. The attending determines that the patient has acute appendicitis and prepares to take the student to the O.R. for an appendectomy. The surgeon asks you the following questions regarding the surgery. What nerve is at risk when an incision is made at McBurney's point? A. iliohypogastric and thoracoabdominal nerves B. ilioinguinal and subcostal nerve C. subcostal and iliohypogastric nerve D. thoracoabdominal and subcostal nerve E. iliohypogastric and ilioinguinal nerve

E. inflammation of the fascia transversalis, parietal and visceral peritoneum

A 21 year old female student presents to the emergency room with a complaint of "colicky" periumbilical pain which has intensified over the last 6-8 hours and now has started to migrate to the right lower quadrant. The patient reports some initial nausea, and as the pain has increased she has had increasing emesis and anorexia Physical exam demonstrates the patient has no distension, auscultation reveals hyperactive bowel sounds, and on palpation the patient demonstrates abdominal guarding and rebound tenderness, and the muscles of the anterior wall in the right lower quadrant are rigid In addition, the patient has low grade fever, and laboratory tests reveal a rising white blood cell count. The attending determines that the patient has acute appendicitis and prepares to take the student to the O.R. for an appendectomy. The surgeon asks you the following questions regarding the surgery. Why were the muscles of the anterior abdominal wall rigid? A. inflammation of the visceral and parietal peritoneum B. inflammation of the visceral peritoneum C. inflammation of the fascia transversalis D. inflammation of the parietal peritoneum E. inflammation of the fascia transversalis, parietal and visceral peritoneum

D. Passes through the deep inguinal ring The genitofemoral nerve descends on the anterior surface of the psoas muscle and gives rise to a genital branch, which enters the inguinal canal through the deep inguinal ring to supply the cremaster muscle, and a femoral branch, which supplies the skin of the femoral triangle. The genitofemoral nerve is not a branch of the femoral nerve but arises from the lumbar plexus and does not supply the testis. It is the ilioinguinal nerve that gives rise to an anterior scrotal branch

A 21-year-old man developed a hernia after lifting heavy boxes while moving into his new apartment. During the repair of his resulting hernia, the urologist recalls that the genitofemoral nerve: A. Runs in front of the quadratus lumborum B. Is a branch of the femoral nerve C. Supplies the testis D. Passes through the deep inguinal ring E. Gives rise to an anterior scrotal branch

D. Ascending and transverse colons The ascending and transverse colons receive blood from the superior mesenteric artery and parasympathetic nerve fibers from the vagus nerve. However, the descending and sigmoid colons receive blood from the inferior mesenteric artery and the parasympathetic nerve fibers from the pelvic splanchnic nerve arising from sacral spinal nerves (S2-S4).

A 21-year-old man suffers a penetrating knife wound in the abdomen which severs both the superior mesenteric artery and the vagus nerve. Which portion of the colon would most likely be impaired by this injury? A. Ascending and descending colons B. Transverse and sigmoid colons C. Descending and sigmoid colons D. Ascending and transverse colons E. Transverse and descending colons

C. Dilation of the bronchial lumen The parasympathetic nerve fibers in the vagus nerve constrict the bronchial lumen, contract bronchial smooth muscle, stimulate bronchial gland secretion, decrease heart rate, and constrict the coronary artery. The vagus nerve also carries afferent fibers of pain, cough reflex, and stretch of the lung (during inspiration).

A 21-year-old patient with a stab wound reveals a laceration of the right vagus nerve proximal to the origin of the recurrent laryngeal nerve. Which of the following conditions would most likely result from this lesion? A. Contraction of bronchial muscle B. Stimulation of bronchial gland secretion C. Dilation of the bronchial lumen D. Decrease in cardiac rate E. Constriction of coronary artery

B. Fascia

A 22-year-old man presented to his family physician with a laceration of the fibrous sheets or bands that cover his body under the skin and surround the muscles. Which of the following structures would most likely be injured? A. Tendon B. Fascia C. Synovial tendon sheath D. Aponeurosis E. Ligament

D. Epidural space The space between the vertebral canal and the dura mater is the epidural space, which contains the internal vertebral venous plexus. The spinal cord and blood vessels lie deep to the pia mater. The space between the arachnoid and the dura maters is the subdural space, which contains a film of fluid. The subarachnoid space contains CSF.

A 23-year-old jockey falls from her horse and complains of headache, backache, and weakness. Radiologic examination would reveal blood in which of the following spaces if the internal vertebral venous plexus was ruptured? A. Space deep to the pia mater B. Space between the arachnoid and the dura maters C. Subdural space D. Epidural space E. Subarachnoid space

C. VA and preganglionic sympathetic fibers The greater splanchnic nerves contain VA and preganglionic sympathetic visceral efferent (VE) fibers.

A 23-year-old man suffered a gunshot wound, and his greater splanchnic nerve was destroyed. Which of the following nerve fibers would be injured? A. Somatic afferent (SA) and preganglionic sympathetic fibers B. Visceral afferent (VA) and postganglionic sympathetic fibers C. VA and preganglionic sympathetic fibers D. Somatic efferent (SE) and postganglionic sympathetic fibers E. VA and SE fibers

B. Sacral hiatus Caudal (epidural) anesthesia is used to block the spinal nerves in the epidural space by injecting local anesthetic agents via the sacral hiatus located between the sacral cornua. An intervertebral foramen transmits the dorsal and ventral primary rami of the spinal nerves. The vertebral canal accommodates the spinal cord. Dorsal and ventral sacral foramina transmit the dorsal and ventral primary rami of the sacral nerves.

A 24-year-old woman comes to the hospital to deliver her baby. Her obstetrician uses a caudal anesthesia during labor and childbirth to block the spinal nerves in the epidural space. Local anesthetic agents are most likely injected via which of the following openings? A. Intervertebral foramen B. Sacral hiatus C. Vertebral canal D. Dorsal sacral foramen E. Ventral sacral foramen

B. Rhomboid major The rhomboid major is a superficial muscle of the back; it is innervated by the dorsal scapular nerve, which arises from the ventral primary ramus of the fifth cervical nerve, and adducts the scapula. The semispinalis capitis, multifidus, rotator longus, and iliocostalis muscles are deep muscles of the back, are innervated by dorsal primary rami of the spinal nerves, and have no attachment to the scapula.

A 25-year-old man with congenital abnormalities at birth has a lesion of the dorsal scapular nerve, making him unable to adduct his scapula. Which of the following muscles is most likely paralyzed? A. Semispinalis capitis B. Rhomboid major C. Multifidus D. Rotator longus E. Iliocostalis

D. Ventral roots of the sacral spinal nerves The cauda equina is the collection of dorsal and ventral roots of the lower lumbar and sacral spinal nerves below the spinal cord. Dorsal and ventral primary rami and dorsal roots of the thoracic spinal nerves and lumbar spinal nerves do not participate in the formation of the cauda equina.

A 25-year-old soldier suffers a gunshot wound on the lower part of his back and is unable to move his legs. A neurologic examination and MRI scan reveal injury of the cauda equina. Which of the following is most likely damaged? A. Dorsal primary rami B. Ventral primary rami C. Dorsal roots of the thoracic spinal nerves D. Ventral roots of the sacral spinal nerves E. Lumbar spinal nerves

E. Rotation The atlantoaxial joints are synovial joints that consist of two plane joints and one pivot joint and are involved primarily in rotation of the head. Other movements do not occur at this joint.

A 26-year-old heavyweight boxer suffered a punch to his mandible, resulting in a slight subluxation (dislocation) of the atlantoaxial joint. The consequence of the injury was decreased range of motion at that joint. What movement would be most affected? A. Extension B. Flexion C. Abduction D. Adduction E. Rotation

B. Would carry impulses away from the cell bodies

A 26-year-old man was stabbed in a bar fight. Axons of the general somatic efferent (GSE) neurons to the shoulder muscles were severed. The damaged axons: A. Would carry impulses toward the cell bodies B. Would carry impulses away from the cell bodies C. Would carry pain impulses D. Are several in numbers for multipolar neurons E. Are found primarily in the gray matter

B. Would carry impulses away from the cell bodies The axons of the neurons carry impulses away from the cell bodies, and dendrites carry impulses to the cell bodies. The axons contain sensory or motor fibers. Multipolar neurons have several dendrites and one axon. The GSE neurons do not carry sensory impulses. The gray matter of the central nervous system consists largely of neuron cell bodies, dendrites, and neuroglia, whereas the white matter consists largely of axons and neuroglia.

A 26-year-old man was stabbed in a bar fight. Axons of the general somatic efferent (GSE) neurons to the shoulder muscles were severed. The damaged axons: A. Would carry impulses toward the cell bodies B. Would carry impulses away from the cell bodies C. Would carry pain impulses D. Are several in numbers for multipolar neurons E. Are found primarily in the gray matter

E. Dorsal pancreatic artery The pancreas is a retroperitoneal organ, except for a small portion of its tail. The dorsal pancreatic artery would be the infected artery because it arises from the splenic artery and runs retroperitoneally along the superior border of the pancreas behind the peritoneum. The other arteries run within layers of the peritoneum. The left gastric arteries run within the lesser omentum; the proper hepatic artery runs within the free margin of the lesser omentum; the middle colic artery runs within the transverse mesocolon; the sigmoid arteries run within the sigmoid mesocolon.

A 26-year-old patient is admitted to a local hospital with a retroperitoneal infection. Which of the following arteries is most likely to be infected? A. Left gastric artery B. Proper hepatic artery C. Middle colic artery D. Sigmoid arteries E. Dorsal pancreatic artery

B. Ampulla of the uterine tube Fertilization occurs in the ampulla of the uterine tube, and a fertilized oocyte forms a blastocyst by day 7 after fertilization and becomes embedded or implanted in the wall of the uterus during the progestational (secretory) phase of the menstrual cycle. Fertilization is the process beginning with the penetration of the secondary oocyte by the sperm and completed by fusion of the male and female pronuclei.

A 26-year-old woman has an amenorrhea, followed by uterine bleeding, pelvic pain, and pelvic mass. Her obstetrician performed a thorough examination, and the patient was diagnosed as having an ectopic pregnancy. Which of the following organs is most likely to provide a normal site of fertilization? A. Fundus of the uterus B. Ampulla of the uterine tube C. Fimbriae D. Infundibulum of the uterine tube E. Body of the uterus

D. In the left fifth intercostal space at the midclavicular line The mitral valve (left AV valve) produces the apical beat (thrust) of the heart, which is most audible over the left fifth intercostal space at the midclavicular line. The pulmonary valve is most audible over the medial end of the second left intercostal space; the aortic valve is most audible over the medial end of the second right intercostal space; and the right AV valve is most audible over the right half of the lower end of the body of the sternum.

A 27-year-old cardiac patient with an irregular heartbeat visits her doctor's office for examination. Where should the physician place the stethoscope to listen to the sound of the mitral valve? A. Over the medial end of the second left intercostal space B. Over the medial end of the second right intercostal space C. In the left fourth intercostal space at the midclavicular line D. In the left fifth intercostal space at the midclavicular line E. Over the right half of the lower end of the body of the sternum

D. Head of the fifth rib The body of vertebra T4 articulates with the heads of the fourth and fifth ribs. The body of the T3 vertebra articulates with the head of the third and fourth ribs. The neck of a rib does not articulate with any part of the vertebra. The transverse process of the vertebra articulates with the tubercle of the corresponding rib. Therefore, the transverse process of vertebra T4 articulates with the tubercle of the fourth rib.

A 27-year-old mountain climber falls from a steep rock wall and is brought to the emergency department. His physical examination and computed tomography (CT) scan reveal dislocation fracture of the upper thoracic vertebrae. The fractured body of the T4 vertebra articulates with which of the following parts of the ribs? A. Head of the third rib B. Neck of the fourth rib C. Tubercle of the fourth rib D. Head of the fifth rib E. Tubercle of the fifth rib

D. Left recurrent laryngeal nerve The left recurrent laryngeal nerve loops around the arch of the aorta near the ligamentum arteriosum, whereas the right recurrent laryngeal nerve hooks around the right subclavian artery. All other nerves are not closely associated with the aortic arch.

A 27-year-old patient with Marfan syndrome has an aneurysm of the aortic arch. This may compress which of the following structures? A. Right vagus nerve B. Left phrenic nerve C. Right sympathetic trunk D. Left recurrent laryngeal nerve E. Left greater splanchnic nerve

C. Lateral horn The lateral horns, which contain sympathetic preganglionic neuron cell bodies, are present between the first thoracic and the second lumbar spinal cord levels (T1-L2). The lateral horns of the second, third, and fourth sacral spinal cord levels (S2-S4) contain parasympathetic preganglionic neuron cell bodies. The entire spinal cord is surrounded by the pia mater and has the dorsal horn, ventral horn, and gray matter. Note that the fourth lumbar spinal cord level is not the same as the fourth vertebral level.

A 27-year-old stuntman is thrown out of his vehicle prematurely when the car used for a particular scene speeds out of control. His spinal cord is crushed at the level of the fourth lumbar spinal segment. Which of the following structures would most likely be spared from destruction? A. Dorsal horn B. Ventral horn C. Lateral horn D. Gray matter E. Pia mater

E. Left and right hepatic veins The right and left hepatic veins drain into the IVC. The right gastroepiploic vein drains into the superior mesenteric vein, but the left one drains into the splenic vein. The right gonadal and suprarenal veins drain into the IVC, whereas the left ones drain into the left renal vein. The right colic vein ends in the superior mesenteric vein, but the left one terminates in the inferior mesenteric vein.

A 27-year-old woman has suffered a gunshot wound to her mid-abdomen. After reviewing the patient's angiogram, a trauma surgeon locates the source of bleeding from pairs of veins that typically terminate in the same vein. Which of the following veins are damaged? A. Left and right ovarian veins B. Left and right gastroepiploic veins C. Left and right colic veins D. Left and right suprarenal veins E. Left and right hepatic veins

B. Lateral horn The lateral horns are found in the gray matter of the spinal cord between T1 and L2 and also between S2 and S4. Therefore, the lateral horns are absent at the L4 spinal cord level.

A 27-year-old woman involved in a car accident is brought into the emergency department. Her magnetic resonance imaging reveals that she has laceration of the spinal cord at the L4 spinal cord level. Which of the following structures would you expect to be intact? A. Dorsal horn B. Lateral horn C. Ventral horn D. Gray matter E. White matter

B. neural tube defect

A 28 year-old G2P1 female patient comes in for a check up for the first time. Sonographic dating puts her roughly around 30 weeks. She states she hasn't been taking any prenatal vitamins or any supplements. On ultrasonography, it has also been identified that the fetus has anencephaly. What is the specific abnormality causing this condition? A. defective neural crest migration B. neural tube defect C. myogenic cell migration defect D. defective division of Somites

C. Dorsal root ganglion Cell bodies of the abdominal pain fibers are located in the dorsal root ganglion. The lateral horn of the spinal cord contains cell bodies of sympathetic preganglionic nerve fibers; the anterior horn contains cell bodies of general somatic efferent (GSE) fibers. The sympathetic chain ganglion contains cell bodies of sympathetic postganglionic fibers, which supply blood vessels, sweat glands, and hair follicles. The celiac ganglion contains cell bodies of sympathetic postganglionic fibers, which supply the visceral organs such as stomach and intestine.

A 29-year-old man comes to a local hospital with a duodenal peptic ulcer and complains of cramping epigastric pain. Which of the following structures harbors the cell bodies of abdominal pain fibers? A. Lateral horn of the spinal cord B. Anterior horn of the spinal cord C. Dorsal root ganglion D. Sympathetic chain ganglion E. Celiac ganglion

B. Suprarenal gland The suprarenal gland is a retroperitoneal organ and is a purely endocrine gland. The pancreas is a retroperitoneal organ and contains endocrine cells, but it is not a purely endocrine gland. The liver and stomach contain endocrine cells, but they are not purely endocrine glands and also are surrounded by peritoneum. The ovary contains endocrine cells and is located in the pelvic cavity.

A 29-year-old woman with abdominal pain was admitted to a local hospital, and examination shows that a retroperitoneal infection is affecting a purely endocrine gland. Which of the following structures is infected? A. Ovary B. Suprarenal gland C. Pancreas D. Liver E. Stomach

C. Ureteric bud The ureteric bud is an outgrowth of the mesonephric duct and develops into the ureter, renal pelvis, calyces, and collecting tubules. However, a bifurcated ureteric bud results in a partial duplication (bifid) of the ureter, whereas two separate ureteric buds result in a complete duplication. Mesonephric duct forms efferent ductules, epididymal duct, ductus deferens, ejaculatory duct, and seminal vesicles. Paramesonephric duct regress and its vestigial remnants form the appendix testis. Metanephros develops into the adult kidney. Pronephros degenerates and never forms functional nephrons.

A 3-year-old boy is admitted to the local children's hospital with complaints of restlessness, abdominal pain, and fever. A magnetic resonance imaging examination reveals that he has a double ureter. Which of the following embryonic structures has most likely failed to develop normally? A. Mesonephric (Wolffian) duct B. Paramesonephric (Müllerian) duct C. Ureteric bud D. Metanephros E. Pronephros

D. Hydrocele If a middle portion of the processus vaginalis persists, it forms a congenital hydrocele. If the entire processus vaginalis persists, it develops a congenital indirect inguinal hernia. Gubernaculum testis is the fetal ligament that connects the bottom of the fetal testis to the developing scrotum. Hematocele is an effusion of blood into the cavity of the tunica vaginalis. Cryptorchidism is failure of the testis to descend from the abdomen to the scrotum.

A 3-year-old boy is diagnosed as having a persistent processus vaginalis in its middle portion. Which condition is most likely to be associated with this developmental anomaly? A. Direct inguinal hernia B. Gubernaculum testis C. Hematocele D. Hydrocele E. Cryptorchidism

D. Is longer than the right primary bronchus The right primary bronchus is shorter than the left one and has a larger diameter. More foreign bodies enter it via the trachea because it is more vertical than the left primary bronchus. The right primary bronchus runs under the arch of the azygos vein and gives rise to the eparterial bronchus.

A 31-year-old man was involved in a severe automobile accident and suffered laceration of the left primary bronchus. The damaged primary bronchus: A. Has a larger diameter than the right primary bronchus B. Often receives more foreign bodies than the right primary bronchus C. Gives rise to the eparterial bronchus D. Is longer than the right primary bronchus E. Runs under the arch of the azygos vein

C. In the left fifth intercostal space On the surface of the chest, the apex of the heart can be located in the left fifth intercostal space slightly medial to the midclavicular (or nipple) line. The sternal angle is located at the level where the second ribs articulate with the sternum. The xiphoid process lies at the level of the T10 vertebra.

A 32-year-old patient who weighs 275 lb comes to the doctor's office. On the surface of the chest, the physician is able to locate the apex of the heart: A. At the level of the sternal angle B. In the left fourth intercostal space C. In the left fifth intercostal space D. In the right fifth intercostal space E. At the level of the xiphoid process of the sternum

A. Above the upper border of the ribs The intercostal veins, arteries, and nerves run in the costal groove beneath the inferior border of the ribs between the internal and the innermost layers of muscles. The transversus thoracis muscles are situated in the internal surface of the lower anterior thoracic wall.

A 32-year-old woman has a tension pneumothorax while kick boxing. It is determined that it can be treated with needle aspiration. To avoid an injury of the intercostal neurovascular bundle, the needle may be inserted in which of the following locations? A. Above the upper border of the ribs B. Deep to the upper border of the ribs C. Beneath the lower border of the ribs D. Between the external and internal intercostals E. Through the transversus thoracis muscle

B. Atlantoaxial joint The atlantoaxial joint is the pivot or trochoid joint. The atlanto-occipital joints are the condyloid (ellipsoidal) joints; the carpometacarpal joint of the thumb is the saddle (sellar) joint; and the proximal tibiofibular joint is the plane (gliding) joint. The intervertebral disk is the secondary cartilaginous (symphysis) joint.

A 33-year-old man slept on his bed in a funny position and now complains of feeling severe pain when he tries to turn his neck. The physician realizes that the problem is in his pivot (trochoid) joint. Which of the following joints would most likely be examined? A. Atlanto-occipital joint B. Atlantoaxial joint C. Carpometacarpal joint D. Proximal tibiofibular joint E. Intervertebral disks

C. Lower intercostal nerves Pain sensation originating from peritoneal irritation by gastric contents in the lesser sac is carried by lower intercostal nerves. The vagus nerves carry sensory fibers associated with reflexes in the GI tract. The greater splanchnic nerves and white rami communicantes carry pain (general visceral afferent [GVA]) fibers from the wall of the stomach and other areas of the GI tract. The gray rami communicantes contains no sensory fibers but contain sympathetic postganglionic fibers.

A 33-year-old man with a perforated gastric ulcer complains of excruciating pain in his stomach. It is observed that the pain comes from peritoneal irritation by gastric contents in the lesser sac. Which of the following nerves contain sensory nerve fibers that convey this sharp, stabbing pain? A. Vagus nerves B. Greater splanchnic nerves C. Lower intercostal nerves D. White rami communicantes E. Gray rami communicantes

E. Lower six posterior The first two posterior intercostal arteries are branches of the highest (superior) intercostal artery of the costocervical trunk; the remaining nine branches are from the thoracic aorta. The internal thoracic artery gives off the upper six anterior intercostal arteries and is divided into the superior epigastric artery and the musculophrenic artery, which gives off anterior intercostal arteries in the 7th, 8th, and 9th intercostal spaces and ends in the 10th intercostal space, where it anastomoses with the deep circumflex iliac artery.

A 33-year-old patient is brought to the cardiac catheterization lab for sudden occlusion at the origin of the descending (thoracic) aorta. This condition would most likely decrease blood flow in which of the following intercostal arteries? A. Upper six anterior B. All of the posterior C. Upper two posterior D. Lower anterior E. Lower six posterior

E. A single left adrenal vein that drains into the left renal vein.

A 34-year-old man is diagnosed with a tumor of the adrenal medulla, on the left, and surgery is indicated to remove the entire gland. During surgery, the vasculature to the left adrenal (suprarenal) gland is ligated. Which of the following must be ligated? A. Two left adrenal veins that drain into the left gonadal vein and the IVC, respectively. B. Three adrenal veins that drain into the left renal, the IVC, and the splenic vein, respectively. C. Three adrenal veins that drain into the left renal, the inferior mesenteric, and the lumbar veins, respectively. D. A single left adrenal vein that drains directly into the IVC. E. A single left adrenal vein that drains into the left renal vein.

D. Iliocostalis The dorsal primary rami of the spinal nerves innervate the deep muscles of the back, including the iliocostalis. The other muscles are the superficial muscles of the back, which are innervated by the ventral primary rami of the spinal nerves.

A 34-year-old woman crashes into a tree during a skiing lesson and is brought to a hospital with multiple injuries that impinge the dorsal primary rami of several spinal nerves. Such lesions could affect which of the following muscles? A. Rhomboid major B. Levator scapulae C. Serratus posterior superior D. Iliocostalis E. Latissimus dorsi

D. Descending colon The descending colon is a retroperitoneal organ. The rest of the organs are surrounded by peritoneum.

A 35-year-old woman comes to a local hospital with acute abdominal pain. On examination, her physician observes that an abdominal infection has spread retroperitoneally. Which of the following structures is most likely affected? A. Stomach B. Transverse colon C. Jejunum D. Descending colon E. Spleen

C. Phrenic nerve The diaphragm receives somatic motor fibers solely from the phrenic nerves. However, the peritoneum on the central part of the diaphragm receives sensory fibers from the phrenic nerve, and the peripheral part of the diaphragm receives such fibers from the lower intercostal nerves. The subcostal nerve supplies the peritoneum inferior to the diaphragm. The vagus and greater splanchnic nerves do not carry pain fibers from the peritoneum.

A 35-year-old woman with a history of cholecystectomy arrives in the emergency department with intractable hiccups most likely caused by an abdominal abscess secondary to surgical infection. Which nerve carries pain sensation caused by irritation of the peritoneum on the central portion of the inferior surface of the diaphragm? A. Vagus nerve B. Lower intercostal nerve C. Phrenic nerve D. Greater splanchnic nerve E. Subcostal nerve

D. Exocrine gland secretion Skin has sweat glands and sebaceous glands, which are exocrine glands. Skin produces vitamin D, but it does not produce a trophic hormone and vitamin A. In addition, skin contains no GSE and parasympathetic GVE nerve fibers.

A 36-year-old man suffers a first-degree burn on his neck, arm, and forearm from a house fire. Which of the following skin structures or functions is most likely damaged or impaired? A. GSE nerves B. Parasympathetic general visceral efferent nerves C. Trophic hormone production D. Exocrine gland secretion E. Vitamin A production

E. Cancer in the body of the pancreas Because the bile duct traverses the head of the pancreas, cancer in the head of the pancreas obstructs the bile duct, resulting in jaundice. Aneurysm of the splenic artery, obstruction of the main pancreatic duct, a stomach ulcer, and cancer in the body of the pancreas are not closely associated with the bile duct.

A 36-year-old woman with yellow pigmentation of the skin and sclerae presents to the outpatient clinic. Which of the following conditions most likely is the cause of her obstructive jaundice? A. Aneurysm of the splenic artery B. Perforated ulcer of the stomach C. Obstruction of the main pancreatic duct D. Cancer in the head of the pancreas E. Cancer in the body of the pancreas

B. Body of the second thoracic vertebra The third rib articulates with the body of the sternum, bodies of the second and third thoracic vertebrae, and transverse process of the third thoracic vertebra.

A 37-year-old house painter fell from a ladder and fractured his left third rib and the structures with which it articulated. Which of the following structures would most likely be damaged? A. Manubrium of the sternum B. Body of the second thoracic vertebra C. Spinous process of the third thoracic vertebra D. Body of the fourth thoracic vertebra E. Transverse process of the second thoracic vertebra

D. Left superior intercostal The left superior intercostal vein is formed by the second, third, and fourth posterior intercostal veins and drains into the left brachiocephalic vein. The right superior intercostal vein drains into the azygos vein, which in turn drains into the SVC. The hemiazygos vein drains into the azygos vein, whereas the internal thoracic vein empties into the brachiocephalic vein.

A 37-year-old man is brought to the emergency department complaining of severe chest pain. His angiogram reveals thromboses of both brachiocephalic veins just before entering the SVC. This condition would most likely cause a dilation of which of the following veins? A. Azygos B. Hemiazygos C. Right superior intercostal D. Left superior intercostal E. Internal thoracic

D. Rectus capitis posterior minor The rectus capitis posterior minor arises from the posterior tubercle of the atlas and inserts on the occipital bone below the inferior nuchal line. The alar ligament extends from the apex of the dens to the medial side of the occipital bone. The apical ligament extends from the dens of the axis to the anterior aspect of the foramen magnum of the occipital bone. The semispinalis cervicis arises from the transverse processes and inserts on the spinous processes. The obliquus capitis inferior originates from the spine of the axis and inserts on the transverse process of the atlas.

A 37-year-old man is brought to the emergency department with a crushed second cervical vertebra (axis) that he suffered after a stack of pallets fell on top of him at work. Which of the following structures would be intact after the accident? A. Alar ligament B. Apical ligament C. Semispinalis cervicis muscle D. Rectus capitis posterior minor E. Obliquus capitis inferior

D. Right ventricle The right ventricle forms a large part of the sternocostal surface of the heart. The left atrium occupies almost the entire posterior surface of the right atrium. The right atrium occupies the right aspect of the heart. The left ventricle lies at the back of the heart and bulges roundly to the left. The base of the heart is formed by the atria, which lie mainly behind the ventricles.

A 37-year-old patient with chronic intermittent palpitation was examined by her physician, and one of the diagnostic studies included a posterior-anterior chest radiograph. Which of the following comprises the largest portion of the sternocostal surface of the heart seen on the radiograph? A. Left atrium B. Right atrium C. Left ventricle D. Right ventricle E. Base of the heart

B. Mitral valve insufficiency The circumflex branch of the left coronary artery supplies the left ventricle, and thus its blockage of blood flow results in necrosis of myocardium in the left ventricle, producing mitral valve insufficiency. The tricuspid valve, AV node, pectinate muscles, and septomarginal trabecula are present in the right atrium and ventricle.

A 37-year-old patient with severe chest pain, shortness of breath, and congestive heart failure was admitted to a local hospital. His coronary angiogram reveals thrombosis in the circumflex branch of the left coronary artery. Which of the following conditions could result from the blockage of blood flow in the circumflex branch? A. Tricuspid valve insufficiency B. Mitral valve insufficiency C. Ischemia of AV node D. Paralysis of pectinate muscle E. Necrosis of septomarginal trabecula

C. Trapezius The trapezius receives blood from the superficial branch of the transverse cervical artery. The latissimus dorsi receives blood from the thoracodorsal artery. The rhomboid major receives blood from the deep or descending branch of the transverse cervical artery. The multifidus and longissimus capitis receive blood from the segmental arteries.

A 38-year-old woman with a long history of shoulder pain is admitted to a hospital for surgery. Which of the following muscles becomes ischemic soon after ligation of the superficial or ascending branch of the transverse cervical artery? A. Latissimus dorsi B. Multifidus C. Trapezius D. Rhomboid major E. Longissimus capitis

A. Greater splanchnic nerve The greater splanchnic nerve carries pain fibers from the upper GI tract. Neither the ventral roots of the spinal nerves nor the gray rami communicantes contain sensory nerve fibers. The vagus nerve contains sensory fibers associated with reflexes, but it does not contain pain fibers. The lower intercostal nerves carry general somatic afferent (GSA) pain fibers from the diaphragm, abdominal wall, and peritoneum, but not GVA pain fibers from the GI tract.

A 38-year-old woman with peptic ulcer disease of the stomach experiences severe abdominal pain. Which of the following nerve structures is most likely involved? A. Greater splanchnic nerve B. Ventral roots of the spinal nerve C. Lower intercostal nerve D. Vagus nerve E. Gray ramus communicans

C. Fifth lumbar nerve A posterolateral herniation of the intervertebral disk at disk levels L4 to L5 affects the fifth lumbar nerve root but rarely affects the fourth lumbar nerve root because of a progressive descending obliquity of the fourth and the fifth lumbar nerve roots. The first seven cervical nerves exit above the corresponding vertebra, and the eighth cervical nerve exits below the seventh cervical vertebra because there are eight cervical nerves but only seven cervical vertebrae. The rest of the spinal nerves exit below their corresponding vertebrae.

A 39-year-old woman with headaches presents to her primary care physician with back pain from a possible herniated disk. Her magnetic resonance imaging (MRI) scan reveals that the posterolateral protrusion of the intervertebral disk between the L4 and L5 vertebrae would most likely affect nerve roots from which of the following spinal nerves? A. Third lumbar nerve B. Fourth lumbar nerve C. Fifth lumbar nerve D. First sacral nerve E. Second sacral nerve

A. Drains bile into the second part of the duodenum The bile duct is formed by the union of the common hepatic and cystic ducts, lies lateral to the proper hepatic artery and anterior to the portal vein in the right free margin of the lesser omentum, traverses the head of the pancreas, and drains bile into the second part of the duodenum at the greater papilla. The endocrine part of the pancreas secretes the hormones insulin and glucagon, which are transported through the bloodstream. The main pancreatic duct carries pancreatic juice containing enzymes secreted from the exocrine part of the pancreas.

A 41-year-old woman is brought to the emergency department by her family because of acute onset of right upper quadrant pain, nausea, and vomiting. Prior to going in to the operating room, the medical student briefly considers reminding the surgeon it is important to remember that the bile duct: A. Drains bile into the second part of the duodenum B. Can be blocked by cancer in the body of the pancreas C. Joins the main pancreatic duct, which carries hormones D. Is formed by union of the right and left hepatic ducts E. Lies posterior to the portal vein in the right free edge of the lesser omentum

D. Splenic vein to the left renal vein Portal hypertension can be reduced by diverting blood from the portal to the caval system. This is accomplished by connecting the splenic vein to the left renal vein or by creating a communication between the portal vein and the IVC. A connection between a hepatic vein and a branch of the portal vein can be accomplished by the TIPS procedure in the treatment of bleeding esophageal varices.

A 42-year-old man with portal hypertension secondary to cirrhosis of the liver and subsequent massive ascites presents to the emergency department. He refuses to have a transjugular intrahepatic portosystemic shunt (TIPS) procedure and prefers surgery. Which of the following surgical connections is involved in the most practical method of shunting portal blood around the liver? A. Superior mesenteric vein to the inferior mesenteric vein B. Portal vein to the superior vena cava C. Portal vein to the left renal vein D. Splenic vein to the left renal vein E. Superior rectal vein to the left colic vein

C. transverse colon

A 42-year-old obese woman with seven children is brought to a local hospital by her daughter. Physical examination and her radiograph reveal that large gallstones have ulcerated through the posterior wall of the fundus of the gallbladder into the intestine. Which of the following parts of the intestine is most likely to contain gallstones? A. cecum B. ascending colon C. transverse colon D. descending colon E. sigmoid colon

C. Transverse colon The fundus of the gallbladder is in contact with the transverse colon, and thus, gallstones erode through the posterior wall of the gallbladder and enter the transverse colon. They are passed naturally to the rectum through the descending colon and sigmoid colon. Gallstones lodged in the body of the gallbladder may ulcerate through the posterior wall of the body of the gallbladder into the duodenum (because the gallbladder body is in contact with the duodenum) and may be held up at the ileocecal junction, producing an intestinal obstruction.

A 42-year-old obese woman with seven children is brought to a local hospital by her daughter. Physical examination and her radiograph reveal that large gallstones have ulcerated through the posterior wall of the fundus of the gallbladder into the intestine. Which of the following parts of the intestine is most likely to initially contain gallstones? A. Cecum B. Ascending colon C. Transverse colon D. Descending colon E. Sigmoid colon

D. Fifth cervical and fourth thoracic nerves All cervical spinal nerves exit through the intervertebral foramina above the corresponding vertebrae except the eighth cervical nerves, which run inferior to the seventh cervical vertebra. All other spinal nerves exit the intervertebral foramina below the corresponding vertebrae. Therefore, the fifth cervical nerve passes between the fourth and the fifth cervical vertebrae, and the fourth thoracic nerve runs between the fourth and the fifth thoracic vertebrae.

A 42-year-old woman with metastatic breast cancer is known to have tumors in the intervertebral foramina between the fourth and fifth cervical vertebrae and between the fourth and fifth thoracic vertebrae. Which of the following spinal nerves may be damaged? A. Fourth cervical and fourth thoracic nerves B. Fifth cervical and fifth thoracic nerves C. Fourth cervical and fifth thoracic nerves D. Fifth cervical and fourth thoracic nerves E. Third cervical and fourth thoracic nerves

A. Elastic tissue in the lungs and thoracic wall Normal, quiet expiration is achieved by contraction of extensible tissue in the lungs and the thoracic wall. The serratus posterior superior muscles, diaphragm, pectoralis major, and serratus anterior are muscles of inspiration.

A 43-year-old female had been lying down on the hospital bed for more than 4 months. Her normal, quiet expiration is achieved by contraction of which of the following structures? A. Elastic tissue in the lungs and thoracic wall B. Serratus posterior superior muscles C. Pectoralis minor muscles D. Serratus anterior muscles E. Diaphragm

C. Third part of the duodenum The third part of the duodenum (transverse portion) crosses anterior to the IVC. The other structures do not cross the IVC anteriorly.

A 43-year-old man complains of abdominal pain just above his umbilicus. On examination, a tumor is found anterior to the IVC. Which of the following structures would most likely be compressed by this tumor? A. Right sympathetic trunk B. Left third lumbar artery C. Third part of the duodenum D. Left renal artery E. Cisterna chyli

C. Splenic artery The splenic artery arises from the celiac trunk, runs along the superior border of the pancreas, and enters the spleen through the lienorenal ligament and the hilus of the spleen. The right gastric artery runs along the lesser curvature of the stomach, and the left gastroepiploic artery runs along the greater curvature of the stomach. The gastroduodenal artery runs behind the first part of the duodenum. The dorsal pancreatic artery descends behind the neck of the pancreas and divides into right and left branches to supply the pancreas.

A 43-year-old woman is admitted to the hospital because of deep abdominal pain in her epigastric region. On examination, it is observed that a retroperitoneal infection erodes an artery that runs along the superior border of the pancreas. Which of the following arteries is likely injured? A. Right gastric artery B. Left gastroepiploic artery C. Splenic artery D. Gastroduodenal artery E. Dorsal pancreatic artery

a. celiac artery

A 44 yr-old accountant develops a bleeding ulcer around tax time. The gastroenterologist visualizes the ulcer in the proximal duodenum. A radiologist has been called to cannulate and embolize the artery supplying the ulcer. Which of the following arteries does the radiologist need to cannulate? a. celiac artery b. SMA c. IMA d. superior epigastric artery

C. Loss of sensation in the fibrous pericardium and mediastinal pleura The phrenic nerve supplies the pericardium, mediastinal and diaphragmatic (central part) pleura, and the diaphragm. It contains SE, SA, and VE (postganglionic sympathetic) fibers. The costal part of the diaphragm receives SA fibers from the intercostal nerves.

A 44-year-old man with a stab wound was brought to the emergency department, and a physician found that the patient was suffering from a laceration of his right phrenic nerve. Which of the following conditions has likely occurred? A. Injury to only somatic efferent (SE) fibers B. Difficulty in expiration C. Loss of sensation in the fibrous pericardium and mediastinal pleura D. Normal function of the diaphragm E. Loss of sensation in the costal part of the diaphragm

D. Internal vertebral venous plexus The internal vertebral venous plexus is located in the spinal epidural space. The vertebral artery and vein occupy the transverse foramina of the upper six cervical vertebrae. The external vertebral venous plexus consists of the anterior part, which lies in front of the vertebral column, and the posterior part, which lies on the vertebral arch. The lumbar cistern is the enlargement of the subarachnoid space between the inferior end of the spinal cord and the inferior end of the subarachnoid space.

A 44-year-old woman comes to her physician and complains of headache and backache. On examination, she is found to have fluid accumulation in the spinal epidural space because of damage to blood vessels or meninges. Which of the following structures has most likely ruptured? A. Vertebral artery B. Vertebral vein C. External vertebral venous plexus D. Internal vertebral venous plexus E. Lumbar cistern

B. Descending aorta The descending (thoracic) aorta is found in the posterior mediastinum. The superior mediastinum contains the trachea and arch of the aorta, and the middle mediastinum contains the ascending aorta, arch of the azygos vein, and main bronchi. The phrenic nerve runs in the middle mediastinum.

A 45-year-old woman presents with a tumor confined to the posterior mediastinum. This could result in compression of which of the following structures? A. Trachea B. Descending aorta C. Arch of the aorta D. Arch of the azygos vein E. Phrenic nerve

C. Pulmonary veins

A 46-year-old male patient with high blood pressure was seen in the emergency department and found to have leakage of blood from the blood vessel that normally carries richly oxygenated blood. Which of the following vessels would most likely be damaged? A. Superior vena cava B. Pulmonary arteries C. Pulmonary veins D. Portal vein E. Coronary sinus

C. Pulmonary veins Pulmonary veins return oxygenated blood to the heart from the lungs. Pulmonary arteries carry deoxygenated blood from the heart to the lungs for oxygen renewal. The portal vein carries deoxygenated blood with nutrients from the intestine to the liver. The superior vena cava and coronary sinus carry deoxygenated blood to the right atrium.

A 46-year-old male patient with high blood pressure was seen in the emergency department and found to have leakage of blood from the blood vessel that normally carries richly oxygenated blood. Which of the following vessels would most likely be damaged? A. Superior vena cava B. Pulmonary arteries C. Pulmonary veins D. Portal vein E. Coronary sinus

B. Arch of the azygos vein The azygos vein arches over the root of the right lung and empties into the SVC. Other veins do not pass over the root of the right lung.

A 46-year-old man comes to his doctor's office and complains of chest pain and headache. His computed tomography (CT) scan reveals a tumor located just superior to the root of the right lung. Blood flow in which of the following veins is most likely blocked by this tumor? A. Hemiazygos vein B. Arch of the azygos vein C. Right subclavian vein D. Right brachiocephalic vein E. Accessory hemiazygos vein

A. Principally to the ipsilateral internal thoracic lymph nodes

A 46-year-old woman consulted her physician about a firm, painless lump in her breast. During physical examination, the physician felt a lump in her upper inner quadrant of the breast. A preliminary diagnosis of carcinoma of the breast was done. The most probable pathway for lymphatic drainage of malignant cells (metastasis) is: A. Principally to the ipsilateral internal thoracic lymph nodes B. Principally to the ipsilateral paratracheal lymph nodes C. Principally to the ipsilateral axillary nodes D. Principally to the contralateral lymph vessels running deep to the pectoralis major E. Cannot be determined with the provided information

A. Preganglionic sympathetic fibers The suprarenal medulla is the only organ that receives preganglionic sympathetic fibers. No other nerve fibers are involved in secretion of norepinephrine from the suprarenal medulla.

A 46-year-old woman is referred to nephrology for evaluation and treatment of severe hypertension. The physician's workup reveals very high levels of norepinephrine from her suprarenal medulla. Which of the following types of nerve fibers are most likely overstimulated? A. Preganglionic sympathetic fibers B. Postganglionic sympathetic fibers C. Somatic motor fibers D. Postganglionic parasympathetic fibers E. Preganglionic parasympathetic fibers

C. Contains the SA node The SA and AV nodes are in the wall of the right atrium and are not associated with the apex of the heart. The oblique cardiac vein drains into the coronary sinus, and the pulmonary veins empty into the left atrium. The right ventricle is hypertrophied by the pulmonary stenosis.

A 47-year-old man with a known history of chronic atrial fibrillation returns to see his cardiologist for follow-up of his cardiac health. The right atrium is important in this case because it: A. Receives blood from the oblique cardiac vein B. Is associated with the apex of the heart C. Contains the SA node D. Receives the right pulmonary vein E. Is hypertrophied by pulmonary stenosis

E. Portal: left gastric vein - Systemic: azygous vein

A 48 year old alcoholic man arrives to the ER vomiting bright red blood. The endoscopy reveals esophageal varices. Which of the following portosystemic anastomosis in the esophageal mucosa is correct? A. Portal: Right gastric vein - Systemic: azygous vein B. Portal: azygous vein - Systemic: left gastric vein C. Portal: left gastric vein - Systemic: hemizygous vein D. Portal: left gastric vein - Systemic: right gastric vein E. Portal: left gastric vein - Systemic: azygous vein

D. Neural crest This patient most likely has melanoma. Melanoma is made up of melanocytes, which originates from neural crest cells of the embryonic ectoderm.

A 49 year-old female presents with 1 cm dark, irregular, slightly raised lesion on her forearm. She is often in the sun working in her garden, and she has noticed it growing bigger in the past few months. What is the embryonic origin of these malignant cells? A. lateral mesoderm B. 3rd phalangeal pouch C. superficial mesoderm D. neural crest E. endoderm

D. Foramen ovale An ASD is a congenital defect in the interatrial septum. During partitioning of the two atria, the opening in the foramen secundum (the foramen ovale) usually closes at birth. If this foramen ovale is not closed completely, this would result in an ASD, shunting blood from the left atrium to the right atrium.

A 5-year-old girl is brought to the emergency department because of difficulty breathing, palpitations, and shortness of breath. Doppler study of the heart reveals an atrial septal defect (ASD). This malformation usually results from incomplete closure of which of the following embryonic structures? A. Ductus arteriosus B. Ductus venosus C. Sinus venarum D. Foramen ovale E. Truncus arteriosus

B. Liver

A 52 year old woman presents with severe upper right quadrant abdominal pain. She is jaundiced, obese and has a history of gallstones. Further examination indicates acute cholecystitis. During the surgery, the doctor inserts her finger into the omental (epiploic) foramen to palpate the structures in the hepatoduodenal ligament. Which of the following structures lies immediately posterior to her finger? A. Bile duct B. Liver C. Inferior Vena Cava D. Abdominal Aorta E. Head of the pancreas

D. The left renal vein runs anterior to both the aorta and the left renal artery The left renal vein runs anterior to both the aorta and the left renal artery. The renal fascia lies external to the perirenal fat and internal to the pararenal fat, and it also surrounds the suprarenal gland. The right renal artery runs behind the IVC and is longer than the left renal artery. Because of the large size of the right lobe of the liver, the right kidney lies a little lower than the left kidney.

A 53-year-old artist with known kidney disease presents to a hospital because her pain has become increasingly more severe. The surgeon performing the kidney operation must remember that: A. The left kidney lies a bit lower than the right one B. The perirenal fat lies external to the renal fascia C. The renal fascia does not surround the suprarenal gland D. The left renal vein runs anterior to both the aorta and the left renal artery E. The right renal artery is shorter than the left renal artery

B. Alveolar (air) sac

A 53-year-old man with a known history of emphysema is evaluated in the emergency department. Laboratory findings and physical examination indicate that the patient is unable to exchange oxygen in the air and carbon dioxide in the blood. This exchange occurs in which portion of the respiratory system? A. Bronchi B. Alveolar (air) sac C. Nasal cavity D. Larynx E. Trachea

A. Superior pancreaticoduodenal artery and inferior pancreaticoduodenal artery The superior pancreaticoduodenal artery is a branch of the gastroduodenal artery, which is a branch of the common hepatic artery, itself a branch of the celiac trunk. The inferior pancreaticoduodenal artery is a branch of the superior mesenteric artery. Occlusion of the celiac trunk would allow blood from the superior mesenteric artery to reach the branches of the celiac trunk via the connections between the superior and inferior pancreaticoduodenal arteries.

A 54 year old man with extensive, severe atherosclerosis sustains a thrombotic occlusion of the celiac trunk. The organs that receive their blood supply from this artery continue to function normally. Anastomoses between which of the following pairs of arteries would explain this phenomenon? A. Superior pancreaticoduodenal artery and inferior pancreaticoduodenal artery B. Right colic artery and middle colic artery C. Proper hepatic artery and gastroduodenal artery D. Left gastroepiploic artery and right gastroepiploic artery E. Left gastric artery and right gastric artery

D. Superior mesenteric artery The uncinate process of the pancreas is a projection of the lower part of its head to the left behind the superior mesenteric vessels. The superior pancreaticoduodenal artery runs between the duodenum and the head of the pancreas. The main pancreatic duct runs transversely through the entire pancreas superior to the uncinate process. The splenic artery runs along the superior border of the pancreas. The portal vein runs behind the neck of the pancreas.

A 54-year-old man comes to a hospital with abdominal pain, jaundice, loss of appetite, and weight loss. On examination of his radiograms and CT scans, a physician finds a slowly growing tumor in the uncinate process of the pancreas. Which of the following structures is most likely compressed by this tumor? A. Main pancreatic duct B. Splenic artery C. Portal vein D. Superior mesenteric artery E. Superior pancreaticoduodenal artery

D. Caput medusae and hemorrhoids are caused by portal hypertension Portal hypertension can cause esophageal varices, caput medusa, and hemorrhoids. The portal vein has higher pressure than systemic veins; the vein and its tributaries have no valves, or, if present, they are insignificant. In addition, the portal vein carries two to three times as much blood as the hepatic artery.

A 54-year-old man with a long history of alcohol abuse presents to the emergency department with rapidly increasing abdominal distention most likely resulting from an alteration in portal systemic blood flow. In his case, which of the following characteristics is associated with the portal vein or the portal venous system? A. Lower blood pressure than in the IVC B. Least risk of venous varices because of portal hypertension C. Distention of the portal vein resulting from its numerous valves D. Caput medusae and hemorrhoids are caused by portal hypertension E. Less blood flow than in the hepatic artery

C. Sinoatrial (SA) node The SA node initiates the impulse of contraction and is known as the pacemaker of the heart. Impulses from the SA node travel through the atrial myocardium to the AV node and then race through the AV bundle (bundle of His), which divides into the right and left bundle branches. The bundle breaks up into terminal conducting fibers (Purkinje fibers) to spread out into the ventricular walls. The moderate band carries the right limb of the AV bundle from the septum to the sternocostal wall of the ventricle.

A 54-year-old patient with amyloidosis and subsequent cardiac dysrhythmias undergoes implantation with an artificial cardiac pacemaker. Which of the following conductive tissues of the heart was defective in function that required the pacemaker? A. Atrioventricular (AV) bundle B. AV node C. Sinoatrial (SA) node D. Purkinje fiber E. Moderator band

C. Anterior interventricular artery The apex of the heart typically receives blood from the anterior interventricular branch of the left coronary artery. The right marginal artery supplies the right inferior margin of the right ventricle; the right coronary artery at its origin supplies the right atrium and ventricle; and the posterior interventricular artery and a circumflex branch of the left coronary artery supply the left ventricle.

A 56-year-old patient recently suffered a myocardial infarction of the apex of the heart. The occlusion by atherosclerosis is in which of the following arteries? A. Marginal artery B. Right coronary artery at its origin C. Anterior interventricular artery D. Posterior interventricular artery E. Circumflex branch of the left coronary artery

C. Chordae tendineae The chordae tendineae are tendinous strands that extend from the papillary muscles to the cusps of the valve. The papillary muscles and chordae tendineae prevent the cusps from being everted into the atrium during ventricular contraction.

A 57-year-old patient has a heart murmur resulting from the inability to maintain constant tension on the cusps of the AV valve. Which of the following structures is most likely damaged? A. Crista terminalis B. Septomarginal trabecula C. Chordae tendineae D. Pectinate muscle E. Annulus fibrosus

E. Left colic vein The left colic vein is a tributary of the inferior mesenteric vein. The middle colic, inferior pancreaticoduodenal, and ileocolic veins drain into the superior mesenteric vein. The left gastroepiploic vein empties into the splenic vein.

A 57-year-old patient has a tumor in the body of the pancreas that obstructs the inferior mesenteric vein just before joining the splenic vein. Which of the following veins is most likely to be enlarged? A. Middle colic vein B. Left gastroepiploic vein C. Inferior pancreaticoduodenal vein D. Ileocolic vein E. Left colic vein

C. Arachnoid villi CSF is absorbed into the venous system primarily through the arachnoid villi projecting into the cranial dural venous sinuses, particularly the superior sagittal sinus. CSF is produced by the choroid plexuses of the ventricles of the brain and is circulated in the subarachnoid space, in which subarachnoid trabeculae are also found. The vertebral venous plexus and internal jugular vein are not involved in the absorption of CSF.

A 57-year-old woman comes into her physician's office complaining of fever, nausea, vomiting, and the worst headache of her life. Tests and physical examination suggest hydrocephalus (widening ventricles) resulting from a decrease in the absorption of CSF. A decrease of flow in the CSF through which of the following structures would be responsible for these findings? A. Choroid plexus B. Vertebral venous plexus C. Arachnoid villi D. Internal jugular vein E. Subarachnoid trabeculae

A. C3-C5 dermatomes

A 58 year old woman complained of recurrent attacks of right upper quadrant pain in her abdomen following fatty meals. In the recent attach, the pain lasted longer than 6 hours and the pain spread over her right shoulder and the tip of her right scapula. A gallstone and acute cholecystitis diagnosis was suspected. The referred pain to the right should is secondary to irritation of the peritoneum on the inferior surface of the diaphragm. The pain pathway is through the A. C3-C5 dermatomes B. C1 dermatome C. C1-C2 dermatomes D. T5-T9 dermatomes E. T10 dermatome

B. Neck of the pancreas The pyloric canal and the neck of the pancreas are situated anterior to the abdominal aorta between the origin of the celiac trunk and the superior mesenteric artery. The transverse colon passes anterior to the superior mesenteric artery and the third part of the duodenum. The other structures are not located in front of the aorta.

A 58-year-old man is admitted to the hospital with severe abdominal pain, nausea, and vomiting, resulting in dehydration. Emergent CT scan reveals a tumor located between the celiac trunk and the superior mesenteric artery. Which structure is likely compressed by this tumor? A. Fundus of the stomach B. Neck of the pancreas C. Transverse colon D. Hepatopancreatic ampulla E. Duodenojejunal junction

A. Left suprarenal vein The veins distal to obstruction are dilated, but the veins proximal to obstruction are not dilated but have low blood pressure. The suprarenal vein drains into the left renal vein and thus is dilated because of high pressure. The right phrenic and right hepatic veins drain into the IVC above the obstruction. The left gastric vein joins the portal vein, which enters the liver.

A 58-year-old man presents with edema of the lower limb and enlarged superficial veins of the abdominal wall. Examination of radiographs and angiograms reveals obstruction of the IVC just proximal to the entrance of the renal vein. This venous blockage may result in dilation of which of the following veins? A. Left suprarenal vein B. Right inferior phrenic vein C. Right hepatic vein D. Left gastric vein E. Portal vein

B. Dorsal root ganglia

A 59-year-old anatomy professor came to her doctor's office for a neurologic examination. Her physician told her that synapses are normally absent in or on which of the following structures? A. Anterior horn of the spinal cord B. Dorsal root ganglia C. Sympathetic chain ganglia D. Dendrites E. Cell bodies

B. Dorsal root ganglia Dorsal root ganglia consist of cell bodies of the unipolar or pseudounipolar neurons and have no synapses. Axosomatic and axodendritic synapses are the most common, but axoaxonic and dendrodendritic contacts are also found in many nerve tissues.

A 59-year-old anatomy professor came to her doctor's office for a neurologic examination. Her physician told her that synapses are normally absent in or on which of the following structures? A. Anterior horn of the spinal cord B. Dorsal root ganglia C. Sympathetic chain ganglia D. Dendrites E. Cell bodies

D. Urachal cyst A urachal cyst or sinus forms from a remnant of the allantois and is found along the midline on a path from the umbilicus to the apex of the urinary bladder. The epithelium lining the cyst produces secretions that gradually fill the remnant with fluid. Very rarely, the entire allantois persists, forming a fistula that is patent from the urinary bladder to the exterior at the umbilicus.

A 6 year old girl presents with a large abdominal mass just superior to the pubic symphysis. The mass is tender when palpated and fixed in location. During surgery, a fluid-filled mass is noted connected to the umbilicus superiorly and to the urinary bladder inferiorly. What is the diagnosis? A. Pelvic kidney B. Horseshoe kidney C. Polycystic disease of kidney D. Urachal cyst E. Exstrophy of the bladder

E. Develops after birth A direct hernia is acquired (develops after birth), whereas an indirect inguinal hernia is congenital. The direct hernia does not enter the deep inguinal ring but occurs through the posterior wall of the inguinal canal, lies medial to the inferior epigastric artery, is covered only by peritoneum, and does not descend into the scrotum.

A 6-year-old boy comes to his pediatrician with a lump and pain in his groin near the thigh. On examination, the physician makes a diagnosis of a direct inguinal hernia because the herniated tissue: A. Enters the deep inguinal ring B. Lies lateral to the inferior epigastric artery C. Is covered by spermatic fasciae D. Descends into the scrotum E. Develops after birth

D. Parasympathetic postganglionic neuron cell bodies Aganglionic megacolon (Hirschsprung disease) is caused by the absence of enteric ganglia (parasympathetic postganglionic neuron cell bodies) in the lower part of the colon, leading to dilatation of the colon proximal to the inactive segment, resulting in an inability to evacuate the bowels. The other neuron cell bodies listed are not involved in this condition.

A 6-year-old girl comes to her pediatrician with constipation, abdominal distention, and vomiting. After thorough workup, she is diagnosed as having Hirschsprung disease (aganglionic megacolon), a congenital disease that leads to dilation of the colon. This condition is caused by an absence of which of the following kinds of neural cell bodies? A. Sympathetic preganglionic neuron cell bodies B. Sympathetic postganglionic neuron cell bodies C. Parasympathetic preganglionic neuron cell bodies D. Parasympathetic postganglionic neuron cell bodies E. Sensory neuron cell bodies

A. Parietal peritoneum and the mesentery

A 60 year old man is admitted to the emergency department with severe abdominal pain. An abdominal CT scan shows a thrombus in an intestinal artery supplying the ileum. Which of the following layers of peritoneum will have to be entered by the surgeon to access the affected vessel? A. Parietal peritoneum and the mesentery B. Parietal peritoneum and the lesser omentum C. Greater and lesser omentum D. Lesser omentum and the gastrosplenic ligament

C. Third part of the duodenum Or could be A. Right third lumbar artery according to complete anatomy)

A 62 year old man complains of abdominal pain just superior to his umbilicus. On CT examination, a mass is found anterior to the IVC at the L3 or L4 level. Which of the following structures would most likely be compressed by this mass at this level? A. Right third lumbar artery B. The portal vein C. Third part of the duodenum D. Right sympathetic trunk E. Left renal artery

D. Left fifth intercostal space To aspirate pericardial fluid, the needle should be inserted into the pericardial cavity through the fifth intercostal space just left of the sternum. Because of the cardiac notch, the needle misses the pleura and lungs, but it penetrates the pericardium. Lung tissues lie beneath the fourth and sixth intercostal spaces.

A 62-year-old patient with pericardial effusion comes to a local hospital for aspiration of pericardial fluid by pericardiocentesis. Which of the following intercostal spaces adjacent to the sternum would be best for insertion of a needle into the pericardial cavity? A. Right fourth intercostal space B. Left fourth intercostal space C. Right fifth intercostal space D. Left fifth intercostal space E. Right sixth intercostal space

B. Right superior intercostal vein The superior intercostal vein is formed by the union of the second, third, and fourth posterior intercostal veins and drains into the azygos vein on the right and the brachiocephalic vein on the left. The azygos vein drains into the SVC. The hemiazygos vein usually drains into the azygos vein.

A 62-year-old woman who is a heavy smoker has an advanced lung cancer that spread into her right third posterior intercostal space posterior to the midaxillary line. If cancer cells are carried in the venous drainage, they would travel first to which of the following veins? A. SVC B. Right superior intercostal vein C. Right brachiocephalic vein D. Azygos vein E. Hemiazygos vein

C. Azygos vein and thoracic duct The aortic hiatus of the diaphragm transmits the azygos vein and thoracic duct. The vagus nerve passes through the esophageal hiatus, and the right phrenic nerve may run through the vena caval hiatus.

A 63-year-old man comes to the emergency department with back pain, weakness, and shortness of breath. On examination, he has an aneurysm of the abdominal aorta at the aortic hiatus of the diaphragm. Which of the following pairs of structures would most likely be compressed? A. Vagus nerve and azygos vein B. Esophagus and vagus nerve C. Azygos vein and thoracic duct D. Thoracic duct and vagus nerve E. Inferior vena cava (IVC) and phrenic nerve

A. Right colic vein The right colic vein belongs to the portal venous system and empties into the superior mesenteric vein, which joins the splenic vein to form the portal vein. The inferior epigastric, inferior phrenic, suprarenal, and ovarian veins belong to the systemic (or caval) venous system and drain directly or indirectly into the IVC.

A 67-year-old woman with a long history of liver cirrhosis was seen in the emergency department. In this patient with portal hypertension, which of the following veins is most likely to be dilated? A. Right colic vein B. Inferior epigastric vein C. Inferior phrenic vein D. Suprarenal vein E. Ovarian vein

E. Kyphosis Kyphosis (hunchback or humpback) is an abnormally increased thoracic curvature, usually resulting from osteoporosis. Lordosis is an abnormal accentuation of the lumbar curvature. Spina bifida occulta is failure of the vertebral arch to fuse (only bony defect). Meningocele is a protrusion of the meninges through the unfused arch of the vertebra, whereas meningomyelocele is a protrusion of the spinal cord and the meninges.

A 69-year-old man has an abnormally increased curvature of the thoracic vertebral column. Which of the following conditions is the most likely diagnosis? A. Lordosis B. Spina bifida occulta C. Meningocele D. Meningomyelocele E. Kyphosis

B. Mesoderm Bronchial cartilages, smooth muscles, and connective tissue are derived from the mesoderm. The bronchial epithelium and glands are derived from the endoderm.

A 7-day-old neonate is diagnosed as having congenital neonatal emphysema, which is caused by collapsed bronchi because of failure of bronchial cartilage development. Bronchial cartilages are derived from which of the following derivations? A. Ectoderm B. Mesoderm C. Endoderm D. Proctodeum E. Neuroectoderm

c. Large intestine (colon) The large intestine absorbs water, salts, and electrolytes. Hence, the patient's diarrhea stems from malabsorption. The stomach mixes food with mucus and gastric juice, which contains hydrochloric acid and enzymes, and forms chyme. The gallbladder receives bile, concentrates it, and stores it. The liver produces bile, whereas the pancreas secretes pancreatic juice, which contains digestive enzymes and releases hormones, such as insulin and glucagon.

A 7-year-old girl comes to the emergency department with severe diarrhea. Tests show that the diarrhea is due to decreased capacity of normal absorption in one of her organs. Which of the following organs is involved? A. Stomach B. Gallbladder C. Large intestine D. Liver E. Pancreas

A. Common bile duct Icteric sclera is the yellowing of the sclera due to the build up of bilirubin (indicative of liver problem?)

A 72 year old male has lost 20 lbs over the last month and presents with upper abdominal pain that radiates to the middle and upper back. During the exam, the doctor notes that the sclerae of his eye are icteric. An abdominal CT scan reveals a tumor within the head of the pancreas. The exocrine secretion of which of the following is being blocked leading directly to icteric sclerae in this patient? A. Common bile duct B. Hepatopancreatic ampulla C. Common hepatic duct D. Left hepatic duct E. Cystic duct

D. The enlarged kidney is irritating the peritoneum lining the psoas major muscle

A 72 year old woman consulted her primary care physican about severe pain on her right flank (side) inferior to her ribs and superior to her hip bone. A radiograph and CT of her kidney revealed a malignant tumor in the inferior aspect of the kidney. Which of the following statements about the pain in her flank, which is increased by hip extension, is most likely correct? A. Metastatic cancer cells from the renal tumor passed via the renal vein to the posterolateral abdominal wall, causing the pain in her flank. B. Stretching of the fibrous capsule of the kidney due to the enlargement of the tumor is compressing the right renal artery. The artery enters the hilum of the kidney at its middle and would not likely be compressed. Cancer cells from the tumor metastasized to the region of the renal nerve plexus via renal lymphatic vessels, resulting in pain in her right side. C. The renal tumor metastasized to the renal pelvis and ureter from which the pain was referred to the flank D. The enlarged kidney is irritating the peritoneum lining the psoas major muscle

D. Superior lobe of the left lung The cardiac notch is a deep indentation of the anterior border of the superior lobe of the left lung. Therefore, the right lung is not involved.

A 75-year-old patient has been suffering from lung cancer located near the cardiac notch, a deep indentation on the lung. Which of the following lobes is most likely to be resected? A. Superior lobe of the right lung B. Middle lobe of the right lung C. Inferior lobe of the right lung D. Superior lobe of the left lung E. Inferior lobe of the left lung

D. Lingula The lingula is the tongue-shaped portion of the upper lobe of the left lung. The right lung has a groove for the horizontal fissure, superior vena cava (SVC), and middle lobe and has a larger capacity than the left lung.

A 75-year-old woman was admitted to a local hospital because a chest radiograph revealed a left lung mass and bronchoscopy revealed lung carcinoma. Which of the following structures or characteristics does the cancerous lung contain? A. Horizontal fissure B. Groove for superior vena cava (SVC) C. Middle lobe D. Lingula E. Larger capacity than the right

D. Dorsal root ganglion

A 76-year-old homicide victim suffered a laceration of the posterior intercostal nerves by a penetrating knife wound. Under microscopy, the pathologist observed numerous degenerated cell bodies of the unipolar or pseudo unipolar neurons. Which of the following structures would most likely provide the abnormal cell morphology? A. Ventral horn of the spinal cord B. Lateral horn of the spinal cord C. Dorsal horn of the spinal cord D. Dorsal root ganglion E. Sympathetic chain ganglion

D. Dorsal root ganglion Ventral, lateral, and dorsal horns and sympathetic chain ganglia contain multipolar neurons, whereas the dorsal root ganglion contains unipolar or pseudounipolar neurons. A laceration of the intercostal nerve injures GSE, postganglionic sympathetic general visceral efferent (GVE), general visceral afferent (GVA), and general somatic afferent (GSA) fibers, whose cell bodies are located in the anterior horn, sympathetic chain ganglia, and dorsal root ganglia.

A 76-year-old homicide victim suffered a laceration of the posterior intercostal nerves by a penetrating knife wound. Under microscopy, the pathologist observed numerous degenerated cell bodies of the unipolar or pseudounipolar neurons. Which of the following structures would most likely provide the abnormal cell morphology? A. Ventral horn of the spinal cord B. Lateral horn of the spinal cord C. Dorsal horn of the spinal cord D. Dorsal root ganglion E. Sympathetic chain ganglion

C. Aorta, inferior phrenic, and renal arteries The suprarenal gland receives arteries from three sources. The superior suprarenal artery arises from the inferior phrenic artery, the middle suprarenal artery arises from the abdominal aorta, and the inferior suprarenal artery arises from the renal artery. The hepatic, superior mesenteric, inferior mesenteric, and splenic arteries do not supply the suprarenal gland.

A 78-year-old man is suffering from ischemia of the suprarenal glands. This condition results from rapid occlusion of direct branches of which of the following arteries? A. Aorta, splenic, and inferior phrenic arteries B. Renal, splenic, and inferior mesenteric arteries C. Aorta, inferior phrenic, and renal arteries D. Superior mesenteric, inferior mesenteric, and renal arteries E. Aorta and hepatic and renal arteries

E. Hemiazygos vein The hemiazygos vein is located in the posterior mediastinum. The brachiocephalic veins, trachea, and arch of the aorta are located in the superior mediastinum, whereas the arch of the azygos vein is found in the middle mediastinum.

A 78-year-old patient presents with advanced cancer in the posterior mediastinum. The surgeons are in a dilemma as to how to manage the condition. Which of the following structures is most likely damaged? A. Brachiocephalic veins B. Trachea C. Arch of the azygos vein D. Arch of the aorta E. Hemiazygos vein

D. Left sixth arch The left sixth aortic arch is responsible for the development of both the ductus arteriosus and the pulmonary arteries. The ductus arteriosus closes functionally in an infant soon after birth, with anatomic closure requiring several weeks

A 9-month-old girl was admitted to the local children's hospital with tachypnea (fast breathing) and shortness of breath. Physical examination further exhibits tachycardia (fast heart rate), a bounding peripheral pulse, and her angiographs reveal a patent ductus arteriosus. Which of the following embryonic arterial structures is most likely responsible for the origin of the patent ductus arteriosus? A. Right fourth arch B. Left fifth arch C. Right fifth arch D. Left sixth arch E. Right sixth arch

C. Iliohypogastric nerve The iliohypogastric nerve runs medially and inferiorly between the internal oblique and transverse abdominal muscles near McBurney point, the point at the junction of the lateral one-third of the line between the anterior superior iliac spine and the umbilicus. The other structures are not found near McBurney point.

A 9-year-old boy was admitted to the emergency department complaining of nausea, vomiting, fever, and loss of appetite. On examination, he was found to have tenderness and pain on the right lower quadrant. Based on signs and symptoms, the diagnosis of acute appendicitis was made. During an appendectomy performed at McBurney point, which of the following structures is most likely to be injured? A. Deep circumflex femoral artery B. Inferior epigastric artery C. Iliohypogastric nerve D. Genitofemoral nerve E. Spermatic cord

D. Ascending colon The right colic and ileocolic arteries arise from the superior mesenteric artery distal to the origin of the middle colic artery. The right colic artery may arise from the ileocolic artery and supplies the ascending colon. The duodenum and pancreas receive blood from the inferior pancreaticoduodenal artery and superior pancreaticoduodenal artery. The pancreas is also supplied by the splenic artery of the celiac trunk. The transverse colon receives blood from the middle colic artery. The descending colon is supplied by the left colic artery, which is a branch of the inferior mesenteric artery.

A 9-year-old girl has crashed into her neighbor's brick wall while riding her bike and is brought to the emergency department with a great deal of abdominal pain. Her radiogram and angiogram show laceration of the superior mesenteric artery immediately distal to the origin of the middle colic artery. If collateral circulation is discounted, which of the following organs may become ischemic? A. Descending colon B. Duodenum C. Pancreas D. Ascending colon E. Transverse colon

Indirect hernia

A CT scan reveals a loop of gut exists the abdominal cavity by passing lateral to the inferior epigastric artery. This type of hernia would be called:

Direct Hernia

A CT scan reveals a loop of gut exits the abdominal cavity by passing medial to the inferior epigastric artery. This type of hernia would be called:

C. Inadequate production of surfactant Verify

A baby born at term is cyanotic and gasping for breath. What problem (more common in preterm infants than in term infants) might account for the symptoms? A. Tracheoesophageal fistula B. Omphalocele C. Inadequate production of surfactant D. Volvulus E. Congenital diaphragmatic hernia

D. Blood flow in coronary arteries is maximal during diastole During diastole, the AV valve is open, and the aortic and pulmonary valves close; whereas during systole, the AV valves close, and the aortic and pulmonary valves open.

A cardiologist is on clinical rounds with her medical students. She asks them, "During the cardiac cycle, which of the following events occurs?" A. AV valves close during diastole B. Aortic valve closes during systole C. Pulmonary valve opens during diastole D. Blood flow in coronary arteries is maximal during diastole E. Aortic valve closes at the same time as AV valve

e. pulmonary trunk

A cardiothoracic surgeon places her finger into the left side of the transverse pericardial sinus and, with her thumb, clamps the vessel lying just in front of her finger. Which vessel has she occluded? a. superior vena cava b. left subclavian artery c. descending aorta d. brachiocephalic trunk e. pulmonary trunk

C. Posterior longitudinal ligament The posterior longitudinal ligament interconnects the vertebral bodies and intervertebral disks posteriorly and runs anterior to the spinal cord within the vertebral canal. The ligamentum nuchae is formed by supraspinous ligaments that extend from the seventh cervical vertebra to the external occipital protuberance and crest. The anterior longitudinal ligament runs anterior to the vertebral bodies. The alar and cruciform ligaments also lie anterior to the spinal cord.

A crush injury of the vertebral column can cause the spinal cord to swell. Which structure would be trapped between the dura and the vertebral body by a swelling spinal cord? A. Anterior longitudinal ligament B. Alar ligament C. Posterior longitudinal ligament D. Cruciform ligament E. Ligamentum nuchae

d. pushes through the inguinal (Hesselbachs) triangle

A direct inguinal hernia: a. is also called a congenital inguinal hernia b. follows the route taken by the testis during development c. courses lateral to the inferior epigastric artery d. pushes through the inguinal triangle e. is the most common type of inguinal hernia

B. Direct inguinal hernia Because it does not go through the inguinal ring

A man lifts a large chest of drawers and as he lifts he feels a sever pain in the lower right quadrant of his abdomen. He finds that he can no longer lift without pain and the next day goes to see his physician. During the surgery the surgeon opens the inguinal region and finds a hernial sac with a small knuckle of intestine projecting through the vessels. The hernia was diagnosed as: A. Congenital inguinal hernia B. Direct inguinal hernia C. Femoral hernia D. Incisional hernia E. Indirect inguinal hernia

C. Denticulate ligament The denticulate ligament is a lateral extension of the pia mater. The filum terminale (internum) is an inferior extension of the pia mater from the tip of the conus medullaris. The coccygeal ligament, which is also called the filum terminale externum or the filum of the dura, extends from the tip of the dural sac to the coccyx. The vascular choroid plexuses produce the CSF in the ventricles of the brain. The tectorial membrane is an upward extension of the posterior longitudinal ligaments from the body of the axis to the basilar part of the occipital bone.

A middle-aged coal miner injures his back after an accidental explosion. His MRI scan reveals that his spinal cord has shifted to the right because the lateral extensions of the pia mater were torn. The function of which of the following structures is most likely impaired? A. Filum terminale internum B. Coccygeal ligament C. Denticulate ligament D. Choroid plexus E. Tectorial membrane

C. Meningomyelocele Meningomyelocele is protrusion of the meninges and spinal cord through the unfused arch of the vertebra. Folic acid in sufficient amounts during pregnancy is shown to prevent these kinds of neural tube defects. Spina bifida occulta is failure of the vertebral arch to fuse (only bony defect). Meningocele is protrusion of the meninges through the defective vertebral arch. Syringomyelocele is protrusion of the meninges and a pathologic tubular cavity in the spinal cord or brain.

A mother brings her young toddler to her pediatrician with new onset of bowel and bladder dysfunction and loss of the lower limb function. Her mother had not taken enough folic acid (to the point of deficiency) during her pregnancy. On examination, the child has protrusion of the spinal cord and meninges and is diagnosed with which of the following conditions? A. Spina bifida occulta B. Meningocele C. Meningomyelocele D. Myeloschisis E. Syringomyelocele

C. Ductus arteriosus A patent ductus arteriosus shunts blood from the pulmonary trunk to the aorta, partially bypassing the lungs, and thus allowing mixed blood to reach the body tissues and causing cyanosis. Dextroposition or transposition of the great arteries must be accompanied by a VSD or a patent ductus arteriosus for the infant to survive. The transposition causes oxygenated blood to pass from the left ventricle into the pulmonary trunk and then into the lungs, but deoxygenated blood travels from the right ventricle into the aorta and then into the systemic circulation.

A mother with diabetes gives birth to a baby who is diagnosed as having dextroposition of the aorta and the pulmonary trunk with cyanosis and shortness of breath. Which of the following structures is required to remain patent until surgical correction of the deformity? A. Umbilical arteries B. Umbilical vein C. Ductus arteriosus D. Ductus venosus E. Sinus venosus

C. Endoderm Cells in the islets of Langerhans, an endocrine portion of the pancreas, are derived from the endoderm of the caudal foregut (from the liver diverticulum). Proctodeum is an invagination of the ectoderm of the terminal part of the hindgut.

A neonatal baby was born with diabetes mellitus due to an inadequate production of insulin. Cells in the endocrine portion of the pancreas that secrete insulin, glucagon, and somatostatin are derived from which of the following? A. Ectoderm B. Mesoderm C. Endoderm D. Proctodeum E. Neural crest cells

A. AP septum failed to develop in a spiral fashion Failure of the aorticopulmonary (AP) septum results in transposition of the great vessels, exhibiting that the aorta is to the right of the pulmonary trunk. Cyanosis is common in transposition of the great vessels. Excessive resorption of the septum primum results in a secundum type of ASD. Pulmonary valve atresia may result in cyanosis, but it will not cause the aorta to be to the right of the pulmonary trunk. A persistent truncus arteriosus is caused by lack of development of the AP septum, resulting in a single outflow track. Coarctation of the aorta is a severe narrowing of the aorta.

A neonate appears severely cyanotic and breathes rapidly. Cardiac echocardiogram reveals the aorta lies to the right of the pulmonary trunk. Which of the following is most likely to have occurred during development? A. AP septum failed to develop in a spiral fashion B. Excessive resorption of septum primum C. Pulmonary valve atresia D. Persistent truncus arteriosus E. Coarctation of the aorta

A. Umbilical hernia In most cases, an umbilical hernia closes spontaneously by age 4 and requires no surgery unless there is incarceration. A symptomatic patent urachus (drainage of urine at the umbilicus) is typically surgically excised. A patent omphalomesenteric duct (Meckel diverticulum) is promptly repaired to minimize the potential for intestinal obstruction or prolapse. Omphalocele and gastroschisis are defects that require surgical repair.

A neonate has a small reducible protrusion through a defined ring at the umbilicus. His pediatrician indicates to the parents that this will likely close spontaneously. Which of the following congenital malformations is present? A. Umbilical hernia B. Symptomatic patent urachus C. Patent omphalomesenteric duct D. Omphalocele E. Gastroschisis

C. Tetralogy of Fallot Tetralogy of Fallot is a combination of congenital cardiac defects consisting of (a) pulmonary stenosis, (b) dextroposition of the aorta (so that it overrides the ventricular septum and receives blood from the right ventricle), (c) ventricular septal defect (VSD), and (d) right ventricular hypertrophy. ASD is a congenital defect in the atrial septum, resulting from a patent foramen ovale. Patent ductus arteriosus shunts blood from the pulmonary trunk to the aorta, bypassing the lungs. Aortic stenosis is an abnormal narrowing of the aortic valve orifice, impeding the blood flow. Coarctation of the aorta is a congenital constriction of the aorta, commonly occurs just distal to the left subclavian artery, causing upper limb hypertension and diminished blood flow to the lower limbs and abdominal viscera.

A newborn baby is readmitted to the hospital with hypoxia and, upon testing, is found to have pulmonary stenosis, dextroposition of the aorta, interventricular septal defect, and hypertrophy of the right ventricle. Which of the following is best described by these symptoms? A. ASD B. Patent ductus arteriosus C. Tetralogy of Fallot D. Aortic stenosis E. Coarctation of the aorta

D. superior vena cava

A patient has a gunshot wound passing directly from anterior to posterior, just lateral to the sternum in the right second intercostal space. The vessel which lies along the path of the bullet is the: A. right subclavian artery B. right internal jugular vein C. ascending aorta D. superior vena cava E. pulmonary trunk

A. Bifurcation of the trachea The sternal angle is the junction of the manubrium and the body of the sternum. It is located at the level where the second rib articulates with the sternum, the trachea bifurcates into the right and left bronchi, and the aortic arch begins and ends. It marks the end of the ascending aorta and the beginning of the descending aorta, and it forms the inferior border of the superior mediastinum.

A patient has a small but solid tumor in the mediastinum, which is confined at the level of the sternal angle. Which of the following structures would most likely be found at this level? A. Bifurcation of the trachea B. Beginning of the ascending aorta C. Middle of the aortic arch D. Articulation of the third rib with the sternum E. Superior border of the superior mediastinum

B. Superior and middle rectal veins Portal-caval anastomoses occur between the left gastric vein and esophageal vein of the azygos, the superior rectal and middle or inferior rectal veins, paraumbilical and superficial epigastric veins, and retrocolic veins and twigs of the renal vein. The hepatic veins and the IVC are systemic or caval veins. The left and right gastric veins belong to the portal venous system. The inferior and superficial epigastric veins and the suprarenal and renal veins are systemic veins.

A patient is rushed to the operating room for an emergent cholecystectomy (resection of a gallbladder) because of cholecystitis. While locating landmarks before surgical resection of an infected gallbladder, the surgeon recalls a portal-caval anastomosis. Which of the following pairs of veins form a portal-caval anastomosis? A. Hepatic veins and IVC B. Superior and middle rectal veins C. Left and right gastric veins D. Inferior and superficial epigastric veins E. Suprarenal and renal veins

C. Unipolar or pseudounipolar neurons Sensation from the skin is carried by GSA fibers, and their cells are unipolar or pseudounipolar types located in the dorsal root ganglia. Multipolar neurons and neurons in the ventral horn and in sympathetic chain ganglia are motor neurons. Bipolar neurons are sensory neurons, but they are not somatic sensory neurons.

A patient presents with a loss of sensation to the skin overlying the shoulder. Injury to which of the following nerve cells would most likely affect the conduction of sensory information to the central nervous system? A. Multipolar neurons B. Bipolar neurons C. Unipolar or pseudounipolar neurons D. Neurons in the ventral horn E. Neurons in sympathetic chain ganglia

D. Drains venous blood into the hepatic veins The liver receives blood from the hepatic artery and portal vein and drains its venous blood into the hepatic veins. The liver manufactures red blood cells in the fetus. The liver plays important roles in bile production and secretion. The quadrate lobe drains bile into the left hepatic duct, not the right hepatic duct, whereas the caudate lobe drains bile into the right and left hepatic ducts. The gallbladder functions to concentrate and store bile.

A patient with cirrhosis is scheduled for a liver transplant. During the operation, the transplant physician explains to his residents that one of the reasons a surgeon must pay close attention to the anatomic location of the liver is that this organ: A. Receives blood only from the hepatic arteries B. Manufactures red blood cells in an adult C. Drains bile from the quadrate lobe into the right hepatic duct D. Drains venous blood into the hepatic veins E. Functions to concentrate and store bile

E. Has teniae coli and epiploic appendages The sigmoid colon has teniae coli and epiploic appendages. The sigmoid colon receives blood from the inferior mesenteric artery, drains its venous blood through the portal tributaries, has its own mesentery (sigmoid mesocolon, therefore, is not a retroperitoneal organ), and receives parasympathetic preganglionic fibers from the pelvic splanchnic nerve.

A patient with diverticulosis of the colon presents for follow-up to his primary care physician with ongoing complaints of left lower quadrant pain and occasionally bloody stools. His physician begins workup recalling that the sigmoid colon: A. Is drained by systemic veins B. Is a retroperitoneal organ C. Receives parasympathetic fibers from the vagus nerve D. Receives its blood from the superior mesenteric artery E. Has teniae coli and epiploic appendages

B. Medial umbilical fold The medial umbilical fold or ligament contains a fibrous remnant of the umbilical artery. The median umbilical fold contains a fibrous remnant of the urachus. The lateral umbilical fold (ligament) contains the inferior epigastric artery and vein, which are adult blood vessels. The ligamentum venosum contains a fibrous remnant of the ductus venosus, and the ligamentum teres hepatic contains a fibrous remnant of the left umbilical vein.

A pediatric surgeon has resected a structure that is a fibrous remnant of an embryonic or fetal artery in a 5-year-old child. Which of the following structures is most likely to be divided? A. Lateral umbilical fold B. Medial umbilical fold C. Median umbilical fold D. Ligamentum teres hepatis E. Ligamentum venosum

C. Left umbilical vein The left umbilical vein becomes the round ligament of the liver after birth. The right umbilical vein did not leave a fibrous remnant because it was degenerated during the early embryonic period. The ductus venosus forms the ligamentum venosum; the ductus arteriosus forms the ligamentum arteriosum; the umbilical artery forms the medial umbilical ligament.

A pediatric surgeon is resecting a possible malignant mass from the liver of a neonate with cerebral palsy. The surgeon divides the round ligament of the liver during surgery. A fibrous remnant of which of the following fetal vessels is severed? A. Ductus venosus B. Ductus arteriosus C. Left umbilical vein D. Right umbilical vein E. Umbilical artery

B. Linea semilunaris The linea semilunaris is a curved line along the lateral border of the rectus abdominis. The linea alba is a tendinous median raphe between the two rectus abdominis muscles. The linea semicircularis is an arcuate line of the rectus sheath, which is the lower limit of the posterior layer of the rectus sheath. The falx inguinalis (conjoint tendon) is formed by aponeuroses of the internal oblique and transverse abdominal muscles (otherwise known as the transversalis fascia).

A physical fitness trainer for a young Hollywood movie star explains the reasons for 100 stomach crunches a day. The young star, a medical student before "hitting it big," reaffirms to his trainer that the lateral margin of the rectus abdominis, the muscle responsible for a washboard stomach, defines which of the following structures? A. Linea alba B. Linea semilunaris C. Linea semicircularis D. Transversalis fascia E. Falx inguinalis

C. axillary region

A physician is performing a breast examination. In addition to the breast tissue on the chest, what other region is critical to complete the palpation of mammary tissue? A. supraclavicular region B. subclavicular region C. axillary region D. parasternal region

E. Left gastroepiploic artery The left gastroepiploic artery runs through the lienogastric ligament, and hence, it is the artery most likely injured. The splenic artery is found in the lienorenal ligament. The right and left gastric arteries run within the lesser omentum. The gastroduodenal artery descends between the duodenum and the head of the pancreas.

A radiograph of a 32-year-old woman reveals a perforation in the posterior wall of the stomach in which the gastric contents have spilled into the lesser sac. The general surgeon has opened the lienogastric (gastrosplenic) ligament to reach the lesser sac and notes the ulcer eroded into an artery. Which artery is most likely involved? A. Splenic artery B. Gastroduodenal artery C. Left gastric artery D. Right gastric artery E. Left gastroepiploic artery

C. Superior Vena Cava A cardiovascular silhouette or cardiac shadow is the contour of the heart and great vessels seen on posterior-anterior chest radiographs. Its right border is formed by the SVC, right atrium, and inferior vena cava; its left border is formed by the aortic arch (aortic knob), pulmonary trunk, left auricle, and left ventricle. The ascending aorta becomes the arch of the aorta and is found in the middle of the heart.

A radiologist reviews the chest radiographs of a 27-year-old victim of a car accident. Which of the following structures forms the right border of the cardiovascular silhouette? A. Arch of the aorta B. Pulmonary trunk C. Superior Vena Cava D. Ascending aorta E. Left ventricle

Caudate lobe of the liver

A surgeon places his finger in the epiploic foramen (of Winslow). Which structure will be superior to his finger?

b. caudate lobe of the liver

A surgeon places his finger in the epiploic foramen (of Winslow). Which structure will be superior to his finger? a. first part of the duodenum b. caudate lobe of the liver c. head of the pancreas d. common bile duct

B. Pubic tubercle

A surgery resident was asked by his attendting to palpate the margin of the superfiical inguinal ring of a healthy female patient. After passing the finger down the edge of the medial crus of the superficial inguinal ring, he felt a bony protuberance deep to the lateral edge, which he correctly identified as: A. Iliopectineal line B. Pubic tubercle C. Pecten pubis D. Iliopubic eminence E. Pubic symphysis

C. ascending aorta and pulmonary trunk

A tape passed through the transverse pericardial sinus would lie behind the A. ascending aorta alone B. ascending aorta and superior vena cava C. ascending aorta and pulmonary trunk D. superior vena cava alone E. superior vena and 4 pulmonary veins

D. Ribs 7 and 9 A thoracentesis is performed for aspiration of fluid in the pleural cavity at or posterior to the midaxillary line, one or two intercostal spaces below the fluid level but not below the ninth intercostal space and, therefore, between ribs 7 and 9. Other intercostal spaces are not preferred.

A thoracentesis is performed to aspirate an abnormal accumulation of fluid in a 37-year-old patient with pleural effusion. A needle should be inserted at the midaxillary line between which of the following two ribs to avoid puncturing the lung? A. Ribs 1 and 3 B. Ribs 3 and 5 C. Ribs 5 and 7 D. Ribs 7 and 9 E. Ribs 9 and 11

A. Medial and lateral The right middle lobar (secondary) bronchus leads to the medial and lateral bronchopulmonary segments. The right superior lobar bronchus divides into the superior, posterior, and anterior segmental (tertiary) bronchi. The right inferior lobar bronchus has the anterior, lateral, posterior, and anterior segmental bronchi.

A thoracic surgeon removed the right middle lobar (secondary) bronchus along with lung tissue from a 57-year-old heavy smoker with lung cancer. Which of the following bronchopulmonary segments must contain cancerous tissues? A. Medial and lateral B. Anterior and posterior C. Anterior basal and medial basal D. Anterior basal and posterior basal E. Lateral basal and posterior basal

C. Genitofemoral nerve Stimulation of the cremaster muscle draws the testis up from the scrotum toward the superficial inguinal ring. The efferent limb of the reflex arc is the genital branch of the genitofemoral nerve, whereas the afferent limb is the femoral branch of the genitofemoral nerve. The other nerves are not involved in the cremasteric reflex.

A young boy is brought to the hospital after a bicycle accident and possible pelvic fracture. While awaiting a computed tomography (CT) scan of his pelvis, the physician's focal neurologic examination reveals a lack of cremasteric reflex, suggesting a nerve has been compromised. Which of the following nerves would be responsible for the impulses in this reflex arch? A. Subcostal nerve B. Lateral femoral cutaneous nerve C. Genitofemoral nerve D. Iliohypogastric nerve E. Femoral nerve

E. He will feel when his bladder is full and can then self catheterize

A young man is brought to the emergency department following a motor vehicle accident, initial examination shows motor weakness ("Spinal shock") in the myotomes for the legs. A computed tomography scan reveals crushed vertebrae that have lesioned his spinal cord of T8. What are the consequences for his bladder function? A. He will not be able to store any urine in his bladder B. He will not be able to control emptying of the bladder C. He will have complete urinary retention D. He will have reflex bladder function E. He will feel when his bladder is full and can then self catheterize

B. Levator scapulae The levator scapulae arise from the transverse processes of the upper cervical vertebrae and insert on the medial border of the scapula. The other muscles are attached to the spinous processes of the vertebrae.

After a 26-year-old man's car was broadsided by a large truck, he is brought to the emergency department with multiple fractures of the transverse processes of the cervical and upper thoracic vertebrae. Which of the following muscles might be affected? A. Trapezius B. Levator scapulae C. Rhomboid major D. Serratus posterior superior E. Rectus capitis posterior major

C. Latissimus dorsi The latissimus dorsi forms boundaries of the auscultation and lumbar triangles and receives blood from the thoracodorsal artery. The levator scapulae, rhomboid minor, and splenius capitis muscles do not form boundaries of these two triangles. The trapezius muscle forms a boundary of the auscultation triangle but not the lumbar triangle. The levator scapulae, rhomboid minor, and trapezius muscles receive blood from the transverse cervical artery. The splenius capitis muscle receives blood from the occipital and transverse cervical arteries.

After an automobile accident, a 47-year-old man suffers an injury to a back muscle that forms the boundaries of the triangle of auscultation, and the lumbar triangle receives no blood. Which of the following muscles might be ischemic? A. Levator scapulae B. Rhomboid minor C. Latissimus dorsi D. Trapezius E. Splenius capitis

D. Muscles in the gastrointestinal (GI) tract

After examination by her neurologist, a patient is told that her parasympathetic nerves are damaged. Which of the following muscles would most likely be affected? A. Muscles in the hair follicles B. Muscles in blood vessels C. Muscles that act at the elbow joint D. Muscles in the gastrointestinal (GI) tract E. Muscles enclosed by epimysium

D. Muscles in the gastrointestinal (GI) tract Smooth muscles in the GI tract are innervated by both parasympathetic and sympathetic nerves. Smooth muscles in the wall of the blood vessels and arrector pili muscles in hair follicles are innervated only by sympathetic nerves. Muscles that act at the elbow joint and muscles enclosed by epimysium are skeletal muscles that are innervated by somatic motor (general somatic efferent [GSE]) nerves.

After examination by her neurologist, a patient is told that her parasympathetic nerves are damaged. Which of the following muscles would most likely be affected? A. Muscles in the hair follicles B. Muscles in blood vessels C. Muscles that act at the elbow joint D. Muscles in the gastrointestinal (GI) tract E. Muscles enclosed by epimysium

E. Septum transversum

After folding of the head region, the structure lying just caudal to the pericardial cavity is the: A. Developing heart B. Connecting stalk C. Primative streak D. Liver E. Septum transversum

E. eighth At the end of the embryonic period, eight weeks post-conception, the essential body structures have formed. The succeeding fetal period is primarily a period of growth and maturation.

All of the essential features of external body form are completed by the end of the ____ week. A. third B. fourth C. fifth D. sixth E. eighth

b. phrenic nerve

All of the following are contents of the posterior mediastinum EXCEPT the: a. thoracic duct b. phrenic nerve c. azygos vein d. esophagus e. right posterior intercostal vein

e. hepatogastric ligament

All of the following are derivates of the dorsal mesogastrium in the embryo, EXCEPT the: a. gastrosplenic ligament b. splenorenal ligament c. phrenicocolic ligament d. gastrocolic ligament e. hepatogastric ligament

B. 2

Almost all of the internal organs are well laid down at _________ months. A. 1 B. 2 C. 3 D. 4 E. 5

E. Muscles of the abdominal wall The abdominal muscles are the muscles of forced expiration, whereas the other distractors are muscles of inspiration.

An 18-year-old girl is thrust into the steering wheel after her car hits the vehicle in front of her and stops suddenly. She then experiences difficulty in (forced) expiration. Which of the following muscles is most likely damaged? A. Levator costarum B. Innermost intercostal muscle C. External intercostal muscle D. Diaphragm E. Muscles of the abdominal wall

D. Anterior cardiac vein The anterior cardiac vein drains into the right atrium. The middle, small, and oblique cardiac veins drain into the coronary sinus. The right and left pulmonary veins drain into the left atrium.

An 8-year-old boy with an ASD presents to his pediatrician. This congenital heart defect shunts blood from the left atrium to the right atrium and causes hypertrophy of the right atrium, right ventricle, and pulmonary trunk. Which of the following veins opens into the hypertrophied atrium? A. Middle cardiac vein B. Small cardiac vein C. Oblique cardiac vein D. Anterior cardiac vein E. Right pulmonary vein

D. Posterior part of the left ventricle The left marginal branch of the circumflex branch of the left coronary artery supplies the posterior portion of the left ventricle. The anterior interventricular artery supplies the anterior aspects of the right and left ventricles and the anterior interventricular septum.

An 83-year-old man with a previously normal coronary circulation is now having symptoms from an embolism of the circumflex branch of the left coronary artery. This condition would result in ischemia of which of the following areas of the heart? A. Anterior part of the left ventricle B. Anterior interventricular region C. Posterior interventricular region D. Posterior part of the left ventricle E. Anterior part of the right ventricle

A. A short septum primum that fails to reach the endocardial cushions.

An atrial septal defect of the ostium primum type is the result of: A. A short septum primum that fails to reach the endocardial cushions. B. Failure of the septum secundum to form C. An abnormally large foramen ovale D. An abnormally small foramen ovale

D. Ligamentum flavum The CSF is located in the subarachnoid space, between the arachnoid layer and the pia mater. In a lumbar puncture, the needle penetrates the skin, fascia, ligamentum flavum, epidural space, dura mater, subdural space, and arachnoid mater. The pia mater forms the internal boundary of the subarachnoid space; thus, it cannot be penetrated by the needle. The posterior longitudinal ligament lies anterior to the spinal cord; thus, it is not penetrated by the needle. The filum terminale externum is the downward prolongation of the spinal dura mater from the second sacral vertebra to the dorsum of the coccyx. The annulus fibrosus consists of concentric layers of fibrous tissue and fibrocartilage surrounding and retaining the nucleus pulposus of the intervertebral disk, which lies anterior to the spinal cord.

An elderly man at a nursing home is known to have degenerative brain disease. When CSF is withdrawn by lumbar puncture for further examination, which of the following structures is most likely penetrated by the needle? A. Pia mater B. Filum terminale externum C. Posterior longitudinal ligament D. Ligamentum flavum E. Annulus fibrosus

D. Superficial inguinal nodes The superficial inguinal lymph nodes receive lymph from the scrotum, penis, buttocks, and lower part of the anal canal, and their efferent vessels enter primarily to the external iliac nodes and ultimately to the lumbar (aortic) nodes. The deep inguinal nodes receive lymph from the testis and upper parts of the vagina and anal canal, and their efferent vessels enter the external iliac nodes.

An elderly man with prostatic hypertrophy returns to his urologist with another case of epididymitis. An acute infection involving the dartos muscle layer of the scrotum most likely leads to an enlargement of which of the following lymph nodes? A. Preaortic nodes B. Lumbar nodes C. External iliac nodes D. Superficial inguinal nodes E. Common iliac nodes

C. Falx inguinalis (conjoint tendon) The falx inguinalis (conjoint tendon) is formed by the aponeuroses of the internal oblique and transverse muscles of the abdomen. The inguinal ligament is formed by aponeurosis of the external oblique abdominal muscle, and the reflected inguinal ligament is formed by certain fibers of the inguinal ligament reflected from the pubic tubercle upward toward the linea alba. The deep inguinal ring lies in the transversalis fascia, and the internal spermatic fascia is formed by the transversalis fascia.

An emergent hernia repair is scheduled. As the attending physician is driving to the hospital, the medical student assisting on the case quickly reviews his anatomy atlas and is trying to commit to memory that the internal oblique abdominis muscle contributes to the formation of which of the following structures? A. Inguinal ligament B. Deep inguinal ring C. Falx inguinalis (conjoint tendon) D. Internal spermatic fascia E. Reflected inguinal ligament

D. Posterior rectus sheath

An obstetrician decides to do a c section to a 45 year old primigravid woman. A transverse suprapubic incision is chosen for that purpose. All of the following abdominal wall layers will be encountered during the incision EXCEPT the: A. Anterior rectus sheath B. Rectus abdominus muscle C. Skin and subcutaneous tissue D. Posterior rectus sheath E. Transversalis fascia, extraperitoneal fat and peritoneum

C. Drains bile into the left hepatic duct The quadrate lobe of the liver drains bile into the left hepatic duct and receives blood from the left hepatic artery. It lies between the gallbladder fossa and the ligamentum teres hepatic, is a medial inferior segment, and is a part of the left lobe.

An oncologist is reviewing a CT scan of a 74-year-old man with newly diagnosed hepatocellular carcinoma. He locates the affected quadrate lobe of the liver that: A. Lies between the IVC and ligamentum venosum B. Receives blood from the right hepatic artery C. Drains bile into the left hepatic duct D. Is a medial superior segment E. Is functionally a part of the right lobe

a. inflammation and distention of the gallbladder and cystic duct

An x-ray shows that a stone has impacted the cystic duct, resulting in intense pain over the superior middle part of the abdomen and extending under the right ribs. The epigastric pain is caused by: a. inflammation and distention of the gallbladder and cystic duct b. inflammation of the left hepatic lobe c. occlusion of the omental foramen d. adhesion of the common hepatic duct to the anterior body wall e. distention of the accessory pancreatic duct

C. is seen more frequently in females than in males is often associated with displacement of medulla and some cerebellum into the vertebral canal. Anencephaly occurs when the cephalic part of the neural tube fails to close. The reflex to swallow amniotic fluid does not develop and excess amniotic fluid, or polyhydramnios, results. It is 4 times more common in females than in males. Neural tube forms from the fusion of neural folds during the 4th week.

Anencephaly: A. is the result of failure of the cephalic part of the neural tube to form B. has its inception during the 5th week of development C. is seen more frequently in females than in males D. is accompanied by a deficiency in amniotic fluid

A. axial skeleton Paraxial mesoderm that forms along the notochord and neural tube divides and forms paired cuboidal bodies called somites starting in week three. They develop in a caudal direction from day 20-30 and give rise to most of axial skeleton (skull base, vertebral column, ribs, and sternum), all skeletal musculature, and the dermis of the skin.

At end of third week, somites begin to develop along the notochord and neural tube. What do they end up forming? A. axial skeleton B. brain C. spinal cord D. sympathetic chain ganglia E. limbs

C. cloacal membrane

At the caudal end of the primitive streak, ectoderm and endoderm fuse as the: A. notochordal canal B. coelom C. cloacal membrane D. neural groove E. notochord

C. Inferior pancreaticoduodenal artery The inferior pancreaticoduodenal artery does not supply the stomach. All of the other arteries supply the stomach. Gastrocolostomy is used to establish a communication between the stomach and colon, bypassing the small intestine when the patient has Crohn disease (inflammation disease) and small bowel obstruction.

Because of an inflammatory bowel disease (Crohn disease) and a small bowel obstruction leading to bowel ischemia, an elderly woman requires bypass of her ileum and jejunum and is scheduled for a gastrocolostomy. The surgeon will ligate all arteries that send branches to the stomach. Which of the following arteries may be spared? A. Splenic artery B. Gastroduodenal artery C. Inferior pancreaticoduodenal artery D. Left gastroepiploic artery E. Proper hepatic artery

True

Both the neurenteric canal and the notochordal plate are present in the conceptus at the same time? True or False

c. spleen

Branches of the superior mesenteric artery typically supply all EXCEPT which of the following? a. duodenum b. pancreas c. spleen d. appendix e. transverse colon

A. Is essential for fertilization

Capacitation of the sperm: A. Is essential for fertilization B. Is caused by the zona pellucida C. Occurs in the male reproductive tract D. Prevents polyspermy E. Removes the head of the sperm

E. amnioblasts

Cells from the primitive streak DO NOT become: A. endoderm B. intermediate mesoderm C. paraxial mesoderm D. lateral plate mesoderm E. amnioblasts

E The right and left bronchial arteries arise from the descending (thoracic) aorta.

Choose the appropriate lettered site or structure in this CT scan (see figure below) of the thorax of a 42-year-old man who complains of chest pain and breathing problems. His electrocardiogram shows left ventricular hypertrophy. From which structure do the bronchial arteries arise?

A The azygos vein drains venous blood into the SVC.

Choose the appropriate lettered site or structure in this CT scan (see figure below) of the thorax of a 42-year-old man who complains of chest pain and breathing problems. His electrocardiogram shows left ventricular hypertrophy. Into which structure does the azygos vein drain venous blood?

B During surgical treatment for cancer in the apex of the right lung by a lobectomy, the right superior secondary (eparterial) bronchus should be removed.

Choose the appropriate lettered site or structure in this CT scan (see figure below) of the thorax of a 42-year-old man who complains of chest pain and breathing problems. His electrocardiogram shows left ventricular hypertrophy. Stenosis of which structure may produce left ventricular hypertrophy?

C The right and left coronary arteries arise from the ascending aorta.

Choose the appropriate lettered site or structure in this CT scan (see figure below) of the thorax of a 42-year-old man who complains of chest pain and breathing problems. His electrocardiogram shows left ventricular hypertrophy. The left coronary artery arises from which structure?

D The left primary bronchus is crossed superiorly by the arch of the aorta and the pulmonary artery.

Choose the appropriate lettered site or structure in this CT scan (see figure below) of the thorax of a 42-year-old man who complains of chest pain and breathing problems. His electrocardiogram shows left ventricular hypertrophy. Which structure is crossed superiorly by the aortic arch and left pulmonary artery?

C Stenosis of the ascending aorta results in left ventricular hypertrophy.

Choose the appropriate lettered site or structure in this CT scan (see figure below) of the thorax of a 42-year-old man who complains of chest pain and breathing problems. His electrocardiogram shows left ventricular hypertrophy. Which structure is most likely to be removed by a surgical oncologist in a surgical resection of a lobe (lobectomy) to remove lung cancer in the apex of the right lung?

C Pulmonary stenosis results in right ventricular hypertrophy.

Choose the appropriate lettered site or structure in this CT scan (see figure) of the thorax. Which structure in this CT scan: Becomes hypertrophied as result of the pulmonary stenosis?

B The right inferior lobar bronchus may be removed in a surgical resection of the inferior lobe of the right lung that is in contact with the diaphragm.

Choose the appropriate lettered site or structure in this CT scan (see figure) of the thorax. Which structure in this CT scan: Can be removed in a surgical resection of a lobe to remove lung cancer on the diaphragmatic surface?

C The right ventricle contains the septomarginal trabecula.

Choose the appropriate lettered site or structure in this CT scan (see figure) of the thorax. Which structure in this CT scan: Contains the septomarginal trabecula (moderator band)?

A The SVC lies on the right side of the ascending aorta and the arch

Choose the appropriate lettered site or structure in this CT scan (see figure) of the thorax. Which structure in this CT scan: Lies on the right side of the aortic arch and ascending aorta?

E The left atrium receives oxygenated blood from the lung by way of the pulmonary veins.

Choose the appropriate lettered site or structure in this CT scan (see figure) of the thorax. Which structure in this CT scan: Receives oxygenated blood via pulmonary veins?

D The ascending aorta takes its origin from the left ventricle and ends at the level of the sternal angle by becoming the arch of the aorta.

Choose the appropriate lettered site or structure in this CT scan (see figure) of the thorax. Which structure in this CT scan: Takes its origin from the left ventricle and ends at the sternal angle?

A The common hepatic artery is divided into the proper hepatic and gastroduodenal arteries.

Choose the appropriate lettered structure in this CT scan of the abdomen (see Figure) at the level of the 12th thoracic vertebra. Which structure is divided into the proper hepatic and gastroduodenal arteries?

E The spleen lies in the left hypochondriac region, is hematopoietic in early life, and later functions in worn-out red blood cell destruction. It filters blood, stores red blood cells, and produces lymphocytes and antibodies.

Choose the appropriate lettered structure in this CT scan of the abdomen (see Figure) at the level of the 12th thoracic vertebra. Which structure is hematopoietic in early life and later destroys worn-out red blood cells?

C The pancreas is an endocrine and exocrine gland; is retroperitoneal in position; and receives blood from the splenic, gastroduodenal, and superior mesenteric arteries.

Choose the appropriate lettered structure in this CT scan of the abdomen (see Figure) at the level of the 12th thoracic vertebra. Which structure is retroperitoneal in position and receives blood from the splenic artery?

B The duodenojejunal flexure is supported by a fibromuscular band called the suspensory ligament of the duodenum (ligament of Treitz), which is attached to the right crus of the diaphragm.

Choose the appropriate lettered structure in this CT scan of the abdomen (see Figure) at the level of the 12th thoracic vertebra. Which structure provides an attachment of the suspensory muscle of the duodenum (ligament of Treitz)?

D The splenic artery is a branch of the celiac trunk, follows a tortuous course along the superior border of the pancreas, and divides into several branches that run through the lienorenal ligament.

Choose the appropriate lettered structure in this CT scan of the abdomen (see Figure) at the level of the 12th thoracic vertebra. Which structure runs along the superior border of the pancreas and enters the lienorenal ligament?

D The superior mesenteric artery, a direct branch of the aorta, supplies blood to the ascending and transverse colons.

Choose the appropriate lettered structure in this CT scan of the abdomen (see figure) at the level of the upper lumbar vertebra. Which structure is a direct branch of the aorta and supplies blood to the ascending and transverse colons?

A The gallbladder receives bile, concentrates it by absorbing water and salt, and stores it.

Choose the appropriate lettered structure in this CT scan of the abdomen (see figure) at the level of the upper lumbar vertebra. Which structure receives bile, concentrates it by absorbing water and salt, and stores it?

E The left renal vein runs anterior to the aorta but posterior to the superior mesenteric artery and receives blood from the gonad and suprarenal gland.

Choose the appropriate lettered structure in this CT scan of the abdomen (see figure) at the level of the upper lumbar vertebra. Which structure receives blood from the left gonad and suprarenal gland?

B The IVC, which receives blood from the liver, kidneys, and other abdominal structures, enters the thorax through the vena caval foramen to empty into the right atrium.

Choose the appropriate lettered structure in this CT scan of the abdomen (see figure) at the level of the upper lumbar vertebra. Which structure receives blood from the liver and kidney and enters the thorax by piercing the central tendon of the diaphragm?

C The liver receives venous blood from the portal vein and arterial blood from the hepatic arteries.

Choose the appropriate lettered structure in this CT scan of the abdomen (see figure) at the level of the upper lumbar vertebra. Which structure receives blood from the portal vein?

D The CSF is found in the lumbar cistern, which is a subarachnoid space in the lumbar area. CSF is produced by vascular choroid plexuses in the ventricles of the brain, circulated in the subarachnoid space, and filtered into the venous system through the arachnoid villi and arachnoid granulations.

Choose the appropriate lettered structure in this MRI scan of the back (see Figure below). CSF is produced by vascular choroid plexuses in the ventricles of the brain and accumulated in which space?

C The cauda equina is formed by a great lash of the dorsal and ventral roots of the lumbar and sacral nerves.

Choose the appropriate lettered structure in this MRI scan of the back (see Figure below). Dorsal and ventral roots of the lower lumbar and sacral nerves are lacerated. Which structure is most likely damaged?

B The conus medullaris is a conical end of the spinal cord and terminates at the level of the L2 vertebra or the intervertebral disk between the L1 and L2 vertebrae. A spinal cord injury at the level of the upper part of the first lumbar vertebra damages the conus medullaris.

Choose the appropriate lettered structure in this MRI scan of the back (see Figure below). The spinal cord is crushed at the level of the upper part of the first lumbar vertebra. Which structure is most likely damaged?

E Epidural fat is shown in the magnetic resonance imaging scan. In addition, the internal vertebral venous plexus lies in the epidural space; thus, venous blood from the plexus may spread into epidural fat.

Choose the appropriate lettered structure in this MRI scan of the back (see Figure below). When the internal vertebral venous plexus is ruptured, venous blood may spread into which tissue and space?

A The intervertebral disk lies between the bodies of two vertebrae and consists of a central mucoid substance, the nucleus pulposus, and a surrounding fibrous tissue and fibrocartilage, the annulus fibrosus. The nucleus pulposus may herniate through the annulus fibrosus, thereby impinging on the roots of the spinal nerves.

Choose the appropriate lettered structure in this MRI scan of the back (see Figure below). Which structure may herniate through the annulus fibrosus, thereby impinging on the roots of the spinal nerve?

C. Septum primum and the fused endocardial cushions

Closure of the foramen primum results from fusion of the: A. Septum secundum and the fused endocardial cushions B. Septum secundum and the septum primum C. Septum primum and the fused endocardial cushions D. Septum primum and the septum spurium E. Septum primum and the sinoatrial valves

A. septum primum and the fused endocardial cushions

Closure of the foramen primum results from fusion of the: A. septum primum and the fused endocardial cushions B. septum primum and the sinoatrial valves C. septum secundum and the septum primum D. septum primum and the septum spurium E. septum secundum and the fused endocardial cushions

Ureteric bud

Collecting ducts of the permanent kidney develop from the:

A. ureteric bud

Collecting ducts of the permanent kidney develop from the: A. ureteric bud B. metanephric blastema (mass) C. mesonephric duct D. splanchnic mesoderm E. mesonephric tubules

D. Absence of vertebrae

Congenital absence of neural crest cells could result in all of the following except: A. Absence of sympathetic chain ganglia B. Facial malformations C. Absence of adrenal medulla D. Absence of vertebrae E. Absence of pigment cells

E. all of the above are correct Neural crest cells form all sensory and autonomic ganglia, Schwann cells, meninges of the brain & spinal cord, adrenal medulla, pigment cells of the skin and connective tissues of the head and neck, including odontoblasts for dentin of the teeth.

Congenital absence of neural crest cells could result in: A. absence of sympathetic chain ganglia B. facial malformations C. absence of adrenal medulla D. absence of pigment cells E. all of the above are correct

E. Vagus (CN X)

Congenital megacolon is a disease that results from a loss of autonomic innervation to the descending colon. Loss of which fibers would cause a loss of gut motility in this segment? A. Pelvic splanchnic nerves B. Preganglionic sympathetic fibers T12-L1 C. Postganglionic sympathetic fibers T12-L1 D. Thoracic splanchnic nerves E. Vagus (CN X)

E. Left atrium and ventricle The left atrium and ventricle receive blood from the circumflex branch of the left coronary artery. The interventricular septum and the apex of the heart are supplied by the anterior interventricular branch of the left coronary artery. The right ventricle receives blood from the anterior interventricular artery and the marginal branch of the right coronary artery. The right atrium receives blood from the right coronary artery.

Coronary angiograms of a 44-year-old man reveal an occlusion of the circumflex branch of the left coronary artery, which leads to myocardial infarction in which of the following areas? A. Right and left ventricles B. Right and left atria C. Interventricular septum D. Apex of the heart E. Left atrium and ventricle

D. Syncytiotrophoblast The cytotrophoblast cells can differentiate into syncytiotrophoblast cells. However, the secondary yolk sac is from endoderm, the amnioblasts are from epiblast, and the exocoelomic membrane is from hypoblast.

Cytotrophoblast cells become: A. Secondary yolk sac B. Amnioblasts C. Exocelomic membrane D. Syncytiotrophoblast

Winged scapula

Damage to the Long Thoracic Nerve could lead to what deformity?

a. left atrium and left ventricle

Damaged heart muscle resulting from occlusion of the circumflex branch of the left coronary artery would most likely be found in which of the following? a. left atrium and left ventricle b. right atrium and right ventricle c. apex of the heart d. right and left ventricles e. left ventricle and posterior interventricular septum

c. inferior mesenteric artery

Derivatives of the hindgut are typically supplied by which of the following? a. celiac trunk b. superior mesenteric artery c. inferior mesenteric artery d. internal iliac artery e. external iliac artery

D. Medial umbilical ligament and the inferior epigastric artery

During a laproscopic examination of the deep surface of the lower anterior abodminal wall the attending physica noted something of intrest and asked the surgery resident to look at the medial inguial fossa. To do so, the young doctor would have to look at the area between the: A. Inferior epigastric artery and urachus B. Medial umbilical ligament and urachus C. Inferior epigastric artery and lateral umbilical fold D. Medial umbilical ligament and the inferior epigastric artery E. Median umbilical ligaments and the medial umbilical ligament

B. Rhomboid major and trapezius The dorsal scapular nerve innervates the levator scapulae and rhomboid muscles, whereas the accessory nerve innervates the trapezius and sternocleidomastoid muscles. The serratus posterior superior is innervated by ventral primary rami of the spinal nerves, whereas the splenius cervicis and erector spinae are innervated by dorsal primary rami of the spinal nerves.

During a schoolyard fight, a 16-year-old boy suffers a deep stab wound near the superior angle of the scapula near the medial border, which compromises both the dorsal scapular and spinal accessory nerves. Such an injury could result in paralysis or weakness of which of the following muscles? A. Trapezius and serratus posterior superior B. Rhomboid major and trapezius C. Rhomboid minor and latissimus dorsi D. Splenius cervicis and sternocleidomastoid E. Levator scapulae and erector spinae

D. Whiplash injury Whiplash injury of the neck is produced by a force that drives the trunk forward, whereas the head lags behind. Scoliosis is a lateral deviation resulting from unequal growth of the spinal column. Hangman fracture is a fracture of the neural arch through the pedicle of the axis that may occur as a result of hanging or motor vehicle accidents. Meningomyelocele is a protrusion of the spinal cord and its meninges. A herniated disk compresses the spinal nerve roots when the nucleus pulposus is protruded through the annulus fibrosus.

During a snowstorm, a 52-year-old man is brought to the emergency department after a multiple car accident. Which of the following conditions is produced by a force that drives the body forward while the head lags behind in a rear-end automobile collision? A. Scoliosis B. Hangman fracture C. Meningomyelocele D. Whiplash injury E. Herniated disk

D. Subarachnoid space Cerebrospinal fluid (CSF) is found in the subarachnoid space, which is a wide interval between the arachnoid layer and the pia mater. The epidural space contains the internal vertebral venous plexus and epidural fat. The subdural space between the arachnoid and the dura contains a little fluid to moisten the meningeal surface. The pia mater closely covers the spinal cord and enmeshes blood vessels on the surfaces of the spinal cord. Thus, the space between the spinal cord and the pia is a potential space.

During an outbreak of meningitis at a local college, a 20-year-old student presents to a hospital emergency department complaining of headache, fever, chills, and stiff neck. He is about to get a lumbar puncture (spinal tap). Cerebrospinal fluid (CSF) is normally withdrawn from which of the following spaces? A. Epidural space B. Subdural space C. Space between the spinal cord and the pia mater D. Subarachnoid space E. Space between the arachnoid and the dura maters

D. Ascending colon The ascending colon is derived from the midgut. The gallbladder and stomach are derived from the foregut, and the descending colon and rectum are derived from the hindgut.

During development, the midgut artery appears to be markedly narrowed at its origin. Which structure is derived from the midgut and may receive inadequate blood supply? A. Gallbladder B. Stomach C. Descending colon D. Ascending colon E. Rectum

B. extends from the prechordal plate to the primitive node

During development, the notochordal process: A. arises from involuting endodermal cells B. extends from the prechordal plate to the primitive node C. is involved in the induction of the primitive gut D. becomes the appendicular skeleton

C. Tracheoesophageal septum The tracheoesophageal septum is formed by the fusion of the tracheoesophageal folds in the midline. This septum divides the foregut into a ventral portion, the laryngotracheal tube (primordium of the larynx, trachea, bronchi, and lungs), and a dorsal portion (primordium of the oropharynx and esophagus).

During early development of the respiratory system, the laryngotracheal tube maintains communication with the primitive foregut. Which of the following embryonic structures is most likely responsible for partitioning these two embryonic structures? A. Tracheoesophageal folds B. Tracheoesophageal fistula C. Tracheoesophageal septum D. Laryngotracheal diverticulum E. Laryngotracheal septum

D. the location of cells in the morula dictates their fate

During human embryonic development: A. the inner cell mass develops into the fetal portion of the placenta B. the yolk sac stores nutrients for the first four weeks of development C. the allantois stores waste products D. the location of cells in the morula dictates their fate

A. Cystic duct, common hepatic duct, and liver

During laparoscopic cholecystectomy the surgeon exposes the cystic artery in the triangle of Calot. What structures form this triangle? A. Cystic duct, common hepatic duct, and liver B. Portal vein, liver and common bile duct C. Gallbladder, liver and common bile duct D. Left hepatic duct, cystic duct and liver E. Right hepatic duct, cystic duct and liver

A. fiber tracts appear to become myelinated about the time they start to function

During maturation of the nervous system: A. fiber tracts appear to become myelinated about the time they start to function B. all myelination of nerve fibers is by oligodendroglial cells C. the first reflexes appear in the caudal region D. there is no movement of the embryo/fetus until the 15th week

A. a premature division of the ureteric bud

During surgery for a benign cyst on the kidney, the surgeon notes that the patients right kidney has two ureters and two renal pelvises. This malformation is: A. a premature division of the ureteric bud B. formation of an extra mass of intermediate mesoderm C. an abnormal division of the pronephros D. an abnormal division of the mesonephros E. a premature division of the metanephric mesoderm

D. Ligamentum teres hepatis The paraumbilical veins and the ligamentum teres hepatis are contained in the free margin of the falciform ligament. The lienorenal ligament contains the splenic vessels and a small portion of the tail of the pancreas. The lienogastric ligament contains the left gastroepiploic and short gastric vessels. The gastrophrenic ligament contains no named structures. The hepatoduodenal ligament, a part of the lesser omentum, contains the bile duct, proper hepatic artery, and portal vein in its free margin.

During surgical treatment of portal hypertension in a 59-year-old man with liver cirrhosis, a surgeon inadvertently lacerates the dilated paraumbilical veins. The veins must be repaired to allow collateral flow. Which of the following ligaments is most likely severed? A. Lienorenal ligament B. Lienogastric ligament C. Gastrophrenic ligament D. Ligamentum teres hepatis E. Ligamentum venosum

G. A, B, and D Statement "C" is NOT correct: there is generally NO mixing of fetal and maternal blood in utero (until birth, during which there is mixing due to rupture of materal and placental vessels). Statement "E" is not correct because the placenta is derived mostly from extraembryonic mesoderm (and some lateral plate mesoderm)

During the development of the placenta: A. cytotrophoblast cells divide and their progeny fuse to form B. syncytiotrophoblast cells. syncytiotrophoblast cells invade the uterine wall. C. fetal blood vessels attach directly to maternal vessels such that there is mixing of fetal and maternal blood. D. the placenta develops from the chorionic plate. E. most of the placenta is derived from embryonic intermediate mesoderm. F. NONE of the above G. A, B, and D

True

During the fourth week in development, the growth of head, tail and lateral folds transform the flat germ disc into a tubular embryo. True or False

A. head

During the period of development of the major organ systems, development and differentiation appear first in what region? A. head B. tail C. mid-embryo D. distal third E. mid-third

A. Syncytiotrophoblast The trophoblast gives rise to both the syncytiotrophoblast and the cytotrophoblast, as well as the extraembryonic mesoderm. The ectoderm is a derivative of the epiblast, and the secondary yolk sac comes from endoderm cells that line the exocoelomic cavity or the primitive yolk sac.

During the second week of development, the trophoblast differentiates into: A. Syncytiotrophoblast B. Ectoderm C. Intraembryonic mesoderm D. Yolk sac E. Allantois

D. epiblast and hypoblast

During the second week, the embryonic disk is composed of: A. ectoderm B. ectoderm and mesoderm C. endoderm D. epiblast and hypoblast E. ectoderm, mesoderm and endoderm

Vitelline veins

During vascular development, the hepatic portal venous system arises from:

C. A yellow and black coated offspring

Embryonic carcinoma (EC) cells were isolated from a yellow-coated mouse with a teratocarcinoma, the EC cells were then micro-injected into the inner cell mass of a blastocyst isolated from a black-coated mouse. The blastocyst was subsequently implanted into the uterus of a while-coated foster mouse. Which of the following would be observed after full-term pregnancy? A. A yellow-coated offspring B. A black-coated offspring C. A yellow and black coated offspring D. A white-coated offspring E. A yellow and white-coated offspring

E. Branches of the portal vein The porta hepatis is the transverse fissure (doorway) in the liver and contains the hepatic ducts, hepatic arteries, and branches of the portal vein. The other structures are not found in the porta hepatis.

Examination of a 54-year-old man reveals an isolated tumor located at the porta hepatis (triad). This tumor most likely compresses which structure? A. Cystic duct B. Hepatic veins C. Common hepatic artery D. Left gastric artery E. Branches of the portal vein

1-2nd

Exposure to teratogens during ________ week (s) usually does not result in congenital malformations; instead, it kills the blastocyst or causes early abortion.

A. 1-2nd

Exposure to teratogens during ________ week (s) usually does not result in congenital malformations; instead, it kills the blastocyts or causes early abortion. A. 1-2nd B. 6th C. 8th D. 10th E. 12th

B. Cytotrophoblast & amnion

Extraembryonic somatic mesoderm is in close association with: A. Syncytiotrophoblast B. Cytotrophoblast & amnion C. Yolk sac D. Syncytiotrophoblast & cytotrophoblast

Fertilization by sperm, initiating embryogenesis

Fetal Landmark: Day 0

Genitalia have male/female differentiations and characteristics

Fetal Landmark: Week 10

Heart begins to beat. Upper and lower limb buds begin to form (7 weeks)

Fetal Landmark: Week 4

Neural tube formed, and Organogenesis begins

Fetal Landmark: Weeks 3-8

Implantation (as a blastocyst)

Fetal Landmark: Within the first week

Bilaminar disk

Fetal Landmark: Within the second week

Gastrulation, primitive streak Notochord and Neural plate begin to form

Fetal Landmark: Within the third week

E. epithelium of lung - mesoderm

Following are four structures together with a germ layer. Identify the INCORRECT association: A. epithelium of pancreas - endoderm B. hair - ectoderm C. bone - mesoderm D. heart - mesoderm E. epithelium of lung - mesoderm

B. paraxial mesoderm

From which embryonic layer does the dura mater arise? A. neural crest B. paraxial mesoderm C. lateral ectoderm D. endoderm

A. third

Gastrulation, the process of formation of the three germ layers. Epiblast cells form a thickening called the primitive streak, with a primitive knot or node located at its cranial end. Epiblast cells invaginate from this streak to form the mesoderm layer and to replace hypoblast with endoderm. The primitive streak first appears at the beginning of the _____ week. A. third B. first C. second D. fifth E. fourth

E. liver parenchyma (functional cells) - endoderm Of those structures mentioned, the adrenal cortex and blood vessels of the stomach are mesodermal. The lung epithelium and liver parenchyma are endodermal, and the olfactory epithelium is ectodermal.

Identify the correct association: A. adrenal cortex - ectoderm B. blood vessels of the stomach - endoderm C. lung epithelium - mesoderm D. olfactory epithelium - brain E. liver parenchyma (functional cells) - endoderm

E. Exstrophy of the bladder The moist, red tissue mass that is exposed to the exterior is actually the posterior wall of the urinary bladder. This is called exstrophy of the bladder and is caused when the anterior abdominal wall and anterior wall of the bladder fail to form. The ureters open onto the posterior wall; therefore, urine drainage is apparent.

Immediately after birth of a boy, a moist, red protrusion of tissue is noted just superior to his pubic symphysis. After observation, urine drainage is noted from the upper lateral corners of this tissue mass. What is the diagnosis? A. Pelvic kidney B. Horseshoe kidney C. Polycystic disease of kidney D. Urachal cyst E. Exstrophy of the bladder

B. Implantation usually occurs about 7 days after fertilization

Implantation of human embryos typically occurs: A. about 1 day after fertilization B. about one week (i.e. 7 days) after fertilization C. about two weeks (i.e. 14 days) after fertilization D. after gastrulation E. at the same time as neural tube closure F. NONE of the above

B. Fundic portion of the stomach

Improper placement of the ligature may lead to damage to the arterial supply of which of the following structures? A. Antrum portion of the stomach B. Fundic portion of the stomach C. Pyloric portion of the stomach D. Head of the pancreas E. Splenic flexure of colon

C. Inner cell mass the cells that line this cavity are called amnioblasts, these cells form the amnion.

In a 7.5 day old blastocyst, the amniotic cavity appears as a slit-like space near the embryonic polar trophoblast and within the: A. Extraembryonic mesoderm B. Exocelomic membrane C. Inner cell mass D. Connecting stalk E. Cytotrophoblast

D. both A and B

In a CT scan of the thorax at the level of the right pulmonary artery, what structure(s) lie(s) directly anterior to this artery? A. superior vena cava B. ascending aorta C. right main bronchus D. both A and B E. A, B and C

C. Splenius capitis The splenius capitis is innervated by dorsal primary rami of the middle and lower cervical nerves. The suboccipital nerve (dorsal primary ramus of C1) supplies the muscles of the suboccipital area, including the rectus capitis posterior major, obliquus capitis superior and inferior, and the semispinalis capitis.

In a freak hunting accident, a 17-year-old boy was shot with an arrow that penetrated into his suboccipital triangle, severing the suboccipital nerve between the vertebral artery and the posterior arch of the atlas. Which of the following muscles would be unaffected by such a lesion? A. Rectus capitis posterior major B. Semispinalis capitis C. Splenius capitis D. Obliquus capitis superior E. Obliquus capitis inferior

A. a branch of the right colic artery must be ligated in this procedure

In surgery to remove a malignant tumor of the ileum that is 20 inches from its terminal end, it is important to know that all of the following are true, EXCEPT: A. a branch of the right colic artery must be ligated in this procedure B. lymph nodes in the superior mesenteric group may contain tumor cells C. the portal vein may have carried tumor cells to the liver D. if the vagal nerve fibers to the cecum are damaged, the cecal lumen will increase in diameter E. several intestinal veins must be ligated in this procedure

A. Muscle connective tissue and blood vessels in the gut wall are derived from the splanchnic mesoderm.

In the development of the gut: A. Muscle connective tissue and blood vessels in the gut wall are derived from the splanchnic mesoderm. B. The celiac trunk represents the blood supply to the midgut. C. The early embryo maintains a connection between the midgut and the yolk sac via the allantois. D. The primitive gut tube is in open communication with the amniotic cavity.

E. all of the above are correct

In the development of the nervous system: A. the neural groove forms from the neural plate B. the neural folds are composed of neuroepithelial cells C. the neural tube maintains temporary contact with the amniotic cavity via neuropores D. neuroepithelial cells give rise to both neuroblasts and gliablasts E. all of the above are correct

D. the inner ear is derived from surface ectoderm

In the development of the nervous system: A. the sulcus limitans is found in the diencephalon B. the infundibulum is an outgrowth of the telencephalon that develops into the neurohypophysis C. sensory nuclei in the hindbrain lie ventral to the sulcus limitans D. the inner ear is derived from surface ectoderm

C. the pontine flexure is in a direction opposite the mesencephalic and cervical flexure There are initially 3 brain vesicles: prosencephalon, mesencephalon & rhombencephalon. The pros- & rhombencephalon divide into 2 parts. The pontine flexure is opposite to the mesencephalic and cervical flexures. The marginal layer contains the nerve fibers arising from the neuroblasts.

In the development of the nervous system: A. there are five primary brain vesicles B. during the 5th week, each brain vesicle subdivides into 2 parts C. the pontine flexure is in a direction opposite the mesencephalic and cervical flexure D. neural epithelial cells are found in the marginal layer

A. mesencephalon - colliculi - visual and auditory reflexes The colliculi serve as relays for auditory and visual reflexes and arise from the mesencephalon. The medulla is derived from myelencephalon. The diencephalon contains the thalamus, a sensory relay station. The cerebellum arises from metencephalon.

In the list below, which embryonic structures are correctly matched with the adult structure and function? A. mesencephalon - colliculi - visual and auditory reflexes B. metencephalon - medulla - reflex center C. telencephalon - thalamus - sensory relay and integration D. telencephalon - cerebellum - motor coordination

C. the esophagus lies to the right of the arch of the aorta A is wrong because:: The brachiocephalic vein lies ANTERIOR to the arteries which originate from the arch of the aorta B is wrong because its the trachea is ANTERIOR to the esophagus (the esophagus lies posterior to the trachea) D is wrong because The RIGHT recurrent laryngeal nerve loops around the subclavian artery the E is wrong because The thoracic duct lies along the LEFT border of the esophagus

In the superior mediastinum: A. the brachiocephalic vein lies posterior to the arteries which originate from the arch of the aorta B. the trachea lies posterior to the esophagus C. the esophagus lies to the right of the arch of the aorta D. the left recurrent laryngeal nerve loops around the subclavian artery the E. thoracic duct lies along the right border of the esophagus

D. the neural plate is induced by the notochordal process and associated mesoderm

In the third week of human embryonic development: A. the amnion appears B. a bilaminar embryonic disc is formed C. the body stalk moves ventrally and joins with the yolk sac stalk to form the umbilical cord D. the neural plate is induced by the notochordal process and associated mesoderm E. the uteroplacental circulation is established

C. Kidneys and gonads Lowkey is the heart as well because the cardiovascular system comes from the mesoderm

Intermediate mesoderm gives rise to the: A. Neural tube B. Heart C. Kidneys and gonads D. Somites E. Notochord

C. intermediate mesoderm The intraembryonic mesoderm differentiates into somites, intermediate mesoderm and lateral plate mesoderm. The neural tube is ectodermal in origin, and the primary germ cells arise from the endoderm of the yolk sac.

Intraembryonic mesoderm differentiates into the: A. yolk sac B. neural tube C. intermediate mesoderm D. primordial germ cells

B. White ramus --- paravertebral ganglion

Its late and you're tired nevertheless you decide to review your notes on the nervous system one last time before going to bed. To your horror, your monster energy drink spills across your notes, washing away some important details of your drawing of a reflex arc. You begin to panic, but then you realize that there is just enough of your drawing of the L1 segment of the spinal cord and its effector organ that you can recrate the details. The axon of the neuron in the picture will travel through the __________ and synapse at the ________ A. Gray ramus --- paravertebral ganglion B. White ramus --- paravertebral ganglion C. Spinal nerve --- terminal ganglion D. Vagus nerve --- terminal ganglion E. Spinal nerve --- effector organ (directly)

A. 5

Lanugo covers the body, some head hairs show, and fetal movements are felt by the mother when the fetus is about how many months old? A. 5 B. 6 C. 7 D. 8 E. 9

A. Thoracic duct The cisterna chyli tapers at its superior aspect and becomes the thoracic duct. Most frequently the cisterna chyli is replaced by a confluence of lymph trunks in the abdominal region. The right lymphatic duct drains the right side of the thorax, right upper extremity, and right side of the neck and head. It usually empties into the right subclavian vein, the internal jugular vein, or the union of the two. The lymphatics of the thoracic cage drain into mediastinal nodes, which in turn drain into either the right lymphatic duct or the thoracic duct.

Lymph from the cisterna chyli will drain into the: A. Thoracic duct B. Right Lymphatic duct C. Cubital nodes D. Inguinal nodes

D. yolk sac development

Major events beginning in the second week of development include: A. notochord differentiation B. somite formation C. angiogenesis D. yolk sac development

Ileum

Meckel's diverticula, vitelline cysts, or vitelline fistulas are most commonly found in association with the:

A. persistence of a portion of the vitelline duct

Meckel's diverticulum is a result of: A. persistence of a portion of the vitelline duct B. improper retraction of herniated intestinal loops C. congenital umbilical hernia D. abnormal rotation of the primitive intestinal loop E. remnants of the allantois

C. migrates cranial to prechordal plate The mesoderm lateral to the notochord arises from invagination of the epiblast at the primitive streak during the third week. Some of this mesoderm forms angiogenic clusters and migrates cranially to reach the cardiogenic area in front of the prochordal plate. Ganglia are from neural crest, which is ectodermal.

Mesoderm lateral to the notochord: A. is derived from hypoblast B. differentiates into ganglia C. migrates cranial to prechordal plate D. induces endoderm differentiation

D. It is a persistent remnant of the embryonic yolk stalk Meckel diverticulum is a persistent remnant of the yolk stalk (vitelline duct) and located 2 ft proximal to the ileocecal junction on the antimesenteric border of the ileum. It is approximately 2 in. long, occurs in approximately 2% of the population, and contains two types of mucosal (gastric and pancreatic) tissues in its wall.

Mrs. Jones is undergoing a routine colonoscopy for colon cancer prevention. The gastroenterologist finds a Meckel diverticulum. Which of the following statements is true about the diverticulum? A. It is found 2 ft distal to the ileocecal junction B. It is located on the mesenteric side of the ileum C. It occurs in approximately 20% of the population D. It is a persistent remnant of the embryonic yolk stalk E. It may contain renal and suprarenal tissues

E. all of the above are correct

Myelination: A. is accomplished by neurilemma or Schwann cells in peripheral nerves B. is accomplished by oligodendroglial cells within the spinal cord C. continues after birth D. is related to function E. all of the above are correct

E. all of the above are correct

Neural crest cells differentiate into: A. postganglionic sympathetic cell bodies B. cells of the inferior mesenteric ganglion C. adrenal medullary cells D. cells of the enteric plexus E. all of the above are correct

B. diencephalon The hypothalamus of the diencephalon is the control center for visceral and endocrine functions.

Of the following, the item most closely associated with the regulation of visceral and endocrine functions is: A. telencephalon B. diencephalon C. mesencephalon D. metencephalon E. myelencephalon

B. epithelium of pharynx

Of the following, which is derived from endoderm? A. hypophysis B. epithelium of pharynx C. epithelium of anal canal D. enamel of the teeth E. muscle

D. tonsils Tonsils arise from endodermal epithelium lining the 2nd pharyngeal pouch. Kidneys & gonads arise from intermediate mesoderm. Hair is ectodermal. Skeletal muscle is from somites and somitomeres. Cardiac & most smooth muscle is from splanchnic mesoderm. Neural crest develops some vascular smooth muscle.

Of the following, which is derived from endoderm? A. muscle B. kidney C. gonads D. tonsils E. hair

Ductus Venosus

Oxygenated blood from the mother enters the fetal circulation via the umbilical vein. Blood passes through which of the following structures immediately before emptying into the inferior vena cava?

B. Vertebrae

Paraxial (somitic) mesoderm give rise to: A. Muscle in the stomach B. Vertebrae C. Muscle of mastication D. Kidney and gonads E. Dorsal root ganglia

E. more than one of the above

Paraxial (somitic) mesoderm gives rise to: A. muscle in the stomach B. vertebrae C. muscles of mastication D. skeletal muscle in the trunk and extremities E. more than one of the above

Vertebrae and ribs

Paraxial (somitic) mesodern gives rise to what?

Prophase I

Primary oocytes are dormant cells in which meiotic phase?

The bones, ligaments, blood vessels, and connective tissue of the limbs.

Somatic mesoderm gives rise to what?

D. Vertebral column Approximately 35 pairs of Somites form. They are derived from a specific subdivision of intraembryonic mesoderm called paraxial mesoderm. Somites differentiate into the components called sclerotome (cartilage and bone of the vertebral column), myotome (epimeric and hypomeric muscle), and dermatome (dermis and subcutaneous area of skin).

Somites may differentiate into which of the following? A. Urogenital ridge B. Kidneys C. Notochord D. Vertebral column E. Epithelial lining of the GI tract

E. all of the above are correct

Somites: A. differentiate into myotomes which give rise to skeletal muscle in trunk and limbs B. differentiate into sclerotomes which give rise to vertebrae C. arise from segmentation of the paraxial mesoderm D. differentiate into myotomes which give rise to skeletal muscle of the limbs E. all of the above are correct

D. have as their primary cause a defect in development of bone

Spina bifida cystica and meningoencephalocele: A. are often caused by a failure of the neural tube to close B. have their inception during the fourth week of development C. always occur together in the affected individual D. have as their primary cause a defect in development of bone

C. a type of tumor containing tissue from all germ layers

Teratomas are: A. the product of teratogens B. malformed fetuses C. a type of tumor containing tissue from all germ layers D. derived from trophoblast E. none of the above

1. Narrowing of the right ventricular outflow region (a pulmonary infundibular stenosis) 2. A Large defect of the interventricular septum 3. An overriding of the aorta that arises directly above septal defect 4. Hypertrophy of right ventricle wall because of the hypertension it causes (higher pressure on right side)

Tetralogy of Fallot causes what four cardiovascular alterations?

F. cytotrophoblast

The ___________ gives rise to the extraembryonic mesoderm. A. ectoderm B. hypoblast C. blastocyst D. trophoblast E. syncytiotrophoblast F. cytotrophoblast G. blastocoele H. inner cell mass I. notochord J. prechordal plate K. primary yolk sac L. amniotic cavity M. intraembryonic coelom N. secondary yolk sac

C. inner cell mass

The amniotic cavity appears as a slit-like space near the embryonic polar trophoblast and within the: A. extraembryonic mesoderm B. exocoelomic membrane C. inner cell mass D. connecting body stalk E. cytotrophoblast

C. between inner cell mass and trophoblast

The amniotic cavity develops: A. on the 10th day B. within the inner cell mass C. between inner cell mass and trophoblast D. in the extraembryonic mesoderm E. between two layers of cytotrophoblast

C. within the inner cell mass near the cytotrophoblast

The amniotic cavity develops: A. on the tenth day B. within the outer cell mass C. within the inner cell mass near the cytotrophoblast D. in extraembryonic mesoderm E. none of the above

Thoracic (descending) aorta

The anterior and posterior intercostal arteries come from what structure?

Internal thoracic artery

The anterior intercostal artery originates from what artery?

Left ventricle

The ascending aorta originates from what part of the heart?

C. Tricuspid and mitral The first heart sound ("lub") is produced by the closure of the tricuspid and mitral valves, whereas the second heart sound ("dub") is produced by the closure of the aortic and pulmonary valves.

The attending physician in the coronary intensive care unit demonstrates to his students a normal heart examination. The first heart sound is produced by near-simultaneous closure of which of the following valves? A. Aortic and tricuspid B. Aortic and pulmonary C. Tricuspid and mitral D. Mitral and pulmonary E. Tricuspid and pulmonary

c. coronary ligament

The bare area of the liver is bounded by the: a. falciform ligament b. hepatoduonal ligament c. coronary ligament d. hepatogastric ligament e. splenorenal ligament

C. forms the embryo proper

The bilaminar germ disc: A. consists of epiblast and mesoblast B. is derived from the outer cells of the morula C. forms the embryo proper D. synthesizes human chorionic gonadotropin, HCG

A. metanephric blastema

The bladder (except the trigone) is derived from tissue associated with the: A. metanephric blastema B. mesonephric ducts C. paramesonephric ducts D. cloaca / urogenital sinus

C. primary yolk sac

The blastocoele becomes the: A. amniotic cavity B. extraembryonic coelom C. primary yolk sac D. chorionic cavity E. secondary cavity

C. Right superior bronchus The eparterial bronchus is the right superior lobar (secondary) bronchus; all of the other bronchi are hyparterial bronchi.

The bronchogram of a 45-year-old female smoker shows the presence of a tumor in the eparterial bronchus. Which airway is most likely blocked? A. Left superior bronchus B. Left inferior bronchus C. Right superior bronchus D. Right middle bronchus E. Right inferior bronchus

B. endoderm of the roof of the yolk sac and embryonic ectoderm

The cloacal membrane consists of: A. embryonic endoderm, mesoderm and ectoderm B. endoderm of the roof of the yolk sac and embryonic ectoderm C. a spherical area of endoderm fused to embryonic mesoderm D. the prechordal plate and the overlying embryonic endoderm E. none of the above

A. Ureteric bud

The collecting tubules of the kidney are derived from the: A. Ureteric bud B. Metanephric blastemal C. Pronephric ducts D. paramesonephric ducts E. mesonephric tubules

B. by about day 11

The completion of interstitial implantation, so that the conceptus is entirely within the endometrium, occurs: A. on about day 20 B. by about day 11 C. by the end of the first week D. by erosion of the myometrial lining E. as a result of endodermal proliferation

Transversalis Fascia

The deep inguinal ring is an opening in what?

d. it transmits the round ligament of the uterus in the female

The deep inguinal ring is best described by which of the following? a. it is a diverticulum of the parietal peritoneum b. it is the site of a direct inguinal hernia c. it is located immediately medial to the inferior epigastric artery d. it transmits the round ligament of the uterus in the female e. it lies just lateral to the rectus sheath at the level of the umbilicus

A. 13th Around the 13th day, the definitive yolk sac is formed by the hypoblast cells that line the inside of the exocoelomic membrane.

The definitive yolk sac of the embryo appears by what day? A. 13th B. 26th C. 72nd D. 94th E. 2nd

A.in the pelvic cavity, wrapped around a right or left common iliac artery

The developing kidneys can sometimes fuse into a single, horseshoe-shaped organ that will usually be found: A.in the pelvic cavity, wrapped around a right or left common iliac artery B.in the abdominal cavity, at the level of the inferior mesenteric artery C.in the abdominal cavity, at the level of the superior mesenteric artery D.in the abdominal cavity, at the level of the celiac artery E.in its normal location in the abdominal cavity, but almost always with supernumerary arteries

Left side when the left pleuroperitoneal membrane remains open

The diaphragmatic hernia is more common in the:

D. All of the above

The distal portion of the bulbus cordis, called the conus cordis, of the embryonic heart: A. Forms the outflow tract (at base of aortic trunk) of the left ventricle. B. Forms the outflow tract (at base of pulmonary trunk) of the right ventricle. C. Becomes subdivided by the proximal portion of the aorticopulmonary septum D. All of the above

B. second month

The embryo is called a fetus after the: A. third week B. second month C. third month D. fourth month E. sixth month

A. sixth arch artery

The embryonic origin of the ligamentum arteriosum is from the: A. sixth arch artery B. second arch artery C. fifth arch artery D. fourth arch artery E. third arch artery

False The epithelium of the gastrointestinal tract is, indeed, endodermal, but the wall of the tract arises from the visceral or splanchnic layer of the lateral plate mesoderm.

The endodermal germ layer gives rise to the epithelium and wall of the gastrointestinal tract. True or False

Endoderm

The epithelial lining of the respiratory system is derived from:

Intercostal artery

The external intercostal muscle receives it blood supply from what artery?

Omphalocele

The failure of the intestines to return to the abdomens at the end of the first stage of rotation of the midgut loop is known as:

A. primitive pulmonary vein

The fetal left atrium is mainly derived from the: A. primitive pulmonary vein B. left horn of the sinus venosus C. right horn of the sinus venosus D. sinus venarum E. primitive atrium

Ligaments teres (hepatic)

The fetal left umbilical vein has an adult intrahepatic remnant known as the

D. Sinus venosus

The fetal right atrium is mainly derived from the: A. Primitive pulmonary veins B. Right pulmonary vein C. Primitive atrium D. Sinus venosus

(A) formation of the primitive streak The formation of the primitive streak on the dorsal surface of the bilaminar embryonic disk is the first indication of gastrulation.

The first indication of gastrulation in the embryo is (A) formation of the primitive streak (B) formation of the notochord (C) formation of the neural tube (D) formation of extraembryonic mesoderm (E) formation of tertiary chorionic villi

E. Distribution of chromatids

The first meiotic division of the primary oocytes is characterized by all of the following EXCEPT: A. Pairing of homologous chromosomes B. DNA replication C. Crossing over D. Production of a polar body E. Distribution of chromatids

B. Its body is widest and nearly vertical Verify

The first rib is considered an atypical rib. This classification is because: A. It has two transversely directed grooves crossing its superior surface B. Its body is widest and nearly vertical C. It is has two facets on their head to articulate with C7 and T1 D. It has no neck E. It has a rough area in its upper surface (tuberosity for serratus anterior)

B. epiblast & hypoblast

The first two intraembryonic germ layers to differentiate are the: A. ectoderm & hypoblast B. epiblast & hypoblast C. ectoderm & endoderm D. ectoderm & mesoderm

C. Septum primum

The floor of the fossa ovalis is derived from: A. Septum secundum B. Endocardial cushion C. Septum primum D. Pulmonary veins E. Sinus venarum

E. Amnioblasts

The following are derived from the primitive streak except: A. Endoderm B. Intermediate mesoderm C. Paraxial mesoderm D. Lateral plate mesoderm E. Amnioblasts

E. suprarenal medulla

The following organs are derived from mesoderm EXCEPT: A. skeletal musculature B. musculature of blood vessels C. cardiac musculature D. suprarenal cortex E. suprarenal medulla

Elevate the ribs during forced inhalation to increase the volume of the thoracic cavity

The function of the external intercostal muscle is to what?

A. ectoderm

The ganglia of the autonomic nervous system are derived from: A. ectoderm B. endoderm C. both D. neither

D. is the motor nerve of the cremaster muscle

The genitofemoral nerve: A. course posterior to the psoas major in the abdominal cavity B. has a genital branch that is vasomotor to the pampiniform plexus C. supplies the quadratus lumborum muscle D. is the motor nerve of the cremaster muscle E. carries postganglionic parasympathetic axons to the penis

D. Dorsal mesentery of the stomach

The greater omentum is derived from the: A. Dorsal mesentery of the colon B. Dorsal mesentery of the small intestine C. Ventral mesentery of the liver D. Dorsal mesentery of the stomach E. Ventral mesentery

C. Arcuate line

The inferior border of the rectus sheath posteriorly is called the? A. Falx inguinalis B. Inguinal ligament C. Arcuate line D. Internal inguinal ring E. External inguinal ring

1. Anterior intercostal artery 2. Superior epigastric artery 3. Musculophrenic artery

The internal thoracic artery branches off into what three arteries

B. Inferior epigastric artery

The internal thoracic artery is sometimes surgically cut near the caudal end of the sternum and used to supply blood to a region of the heart. In these cases, maintenance of adequate blood flow to the rectus abdominis may be dependent on increase flow through which artery? A. Superior epigastric artery B. Inferior epigastric artery C. Umbilical artery D. Superficial circumflex iliac artery E. Deep circumflex iliac artery

Subclavian artery

The internal thoracic artery originates from what artery?

Anterior thoracic wall and breast tissue

The internal thoracic artery provides blood supply to what?

1. Anterior intercostal vein 2. Superior epigastric vein 3. Musculophrenic vein

The internal thoracic vein comes from the union of what three veins?

Subclavian vein

The internal thoracic vein drains into what vein?

B. Intraembryonic coelom

The lateral mesoderm is divided into two distinct layers by the formation of the: A. Extraembryonic coelom B. Intraembryonic coelom C. Cardiogenic region D. Notochord E. Yolk sac

A. inferior epigastric vessels

The lateral umbilical fold contains which of the following structures? A. inferior epigastric vessels B. obliterated inferior epigastric vessels C. obliterated ductus venosus D. obliterated urachus E. obliterated umbilical arteries

C. has a branch which courses in the anterior interventricular sulcus

The left coronary artery: A. is the only artery that supplies the ventricular wall B. accompanies the small cardiac vein C. has a branch which courses in the anterior interventricular sulcus D. usually gives rise to the sinuatrial nodal branch

Arch of the aorta

The left recurrent laryngeal nerve passes around this structure:

A. Septum secundum

The limbus of the fossa ovalis is derived from: A. Septum secundum B. Endocardial cushion C. Septum primum D. Pulmonary veins E. Sinus venarum

N. secondary yolk sac The lining of the gut is derived from the endodermal lining of the secondary yolk sac.

The lining of the _____________ forms the lining of the embryonic gut. A. ectoderm B. hypoblast C. blastocyst D. trophoblast E. syncytiotrophoblast F. cytotrophoblast G. blastocoele H. inner cell mass I. notochord J. prechordal plate K. primary yolk sac L. amniotic cavity M. intraembryonic coelom N. secondary yolk sac

Foregut

The liver forms from the invagination of the:

False The main intercostal vessels run INFERIOR to the border of the rib

The main intercostal vessels (VAN) run in the superior border of the rib? True or False

A. obliterated umbilical arteries

The medial umbilical fold contains which of the following structures? A. obliterated umbilical arteries B. obliterated inferior epigastric vessels C. obliterated ductus venosus D. obliterated urachus E. inferior epigastric vessels

D. costal parts of right and left pleural sacs verify (could be D or C)

The mediastinum is bounded laterally: A. by the parietal layer of serous pericardium B. fibrous pericardium C. mediastinal portions of the right and left pleural sacs D. costal parts of right and left pleural sacs E. right and left lungs

A. is separated into superior and inferior portions of the sternal angle

The mediastinum: A. is separated into superior and inferior portions of the sternal angle B. ends inferiorly at the level of the tracheal bifurcation C. lies anterior to T2 to T10, but not T1, T11, or T12 D. does not contain the thoracic E. duct does not contain the phrenic nerves

a. Mesonephric duct

The metanephric blastema (mass) adjacent to the ureteric bud is derived from the: a. Mesonephric duct b. Intermediate mesoderm c. Urogenital sinus d. Metanephric mesoderm e. Cloaca

E. Uterine tube aka fallopian tube

The most common site for implantation in ectopic pregnancy is: A. Internal Os of the uterus B. Cervix C. Mesentery D. Ovary E. Uterine tube

A. right vitelline vein

The most superior part of the inferior vena cava is derived from: A. right vitelline vein B. left vitelline vein C. left umbilical vein D. right umbilical vein

A. Stomach and duodenal loop

The pancreas lies between the: A. Stomach and duodenal loop B. Liver and stomach C. Duodenal loop and splenic hilum D. Stomach and splenic hilum

E. all of the above are correct

The part of the 13-day embryoblast from which the embryo proper is formed: A. lies between the amniotic cavity and yolk sac B. also contributes to the roof of the yolk sac C. is composed of two primary germ layers D. is attached to the amnion E. all of the above are correct

C. Mesoderm

The podocytes of Bowman's capsule are derived from A. Ectoderm B. Endoderm C. Mesoderm D. Endoderm and Mesoderm E. Neural crest cells

e. supreme superior intercostal artery

The posterior intercostal artery in the right second intercostal space typically arises from which of the following? a. ascending thoracic aorta b. descending thoracic aorta c. right bronchial artery d. internal thoracic artery e. supreme superior intercostal artery

b. middle cardiac vein

The posterior interventricular sulcus contains which of the following? a. small cardiac vein b. middle cardiac vein c. great cardiac vein d. coronary sinus e. oblique vein of the left atrium

A. Arcuate line

The posterior layer of the rectus sheath ends inferiorly at the: A. Arcuate line B. Pectineal line C. Intercrural line D. Linea alba E. Semilunar line

C. third Gastrulation, the process of formation of the three germ layers, occurs during the third week. Epiblast cells form a thickening called the primitive streak, with a primitive knot or node located at its cranial end. Epiblast cells invaginate from this streak to form the mesoderm layer and to replace hypoblast with endoderm.

The primitive streak first appears at the beginning of the ___ week. A. first B. second C. third D. fourth E. fifth

D. is the site of involution of epiblast cells to form mesoderm The primitive streak begins to form on the surface of the epiblast at the beginning of the third week. It is at the primitive streak that epiblast cells invaginate to form the mesoderm, through the process of gastrulation. Streaking was a fad of the 1970's that involved running naked in public, definitely not a winter sport.

The primitive streak: A. is derived from the outer cells of the morula B. is formed during the second week in development C. persists as the cloacal membrane D. is the site of involution of epiblast cells to form mesoderm E. was done in a bathing suit, for those who remember streaking

Yolk sac wall

The primordial germ cells of the human embryo are first observable in the:

A. Yolk sac wall

The primordial germ cells of the human embryo are first observable in the: A. Yolk sac wall B. Genital Ridge C. Indifferent gonad D. Differentiated gonad E. Wall of the allantois

B. Metanephric vesicle

The proximal convoluted tubules of the definitive adult kidney are derived from the A. Ureteric bud B. Metanephric vesicle C. Mesonephric duct D. Mesonephric tubules E. Pronephric tubules

E. Endoderm

The respiratory epithelium arises from A. Somatic mesoderm B. Ectoderm C. Splanchnic mesoderm D. Paraxial mesoderm E. Endoderm

Right subclavian artery

The right recurrent laryngeal nerve passes around this artery:

B. Right subclavian artery

The right recurrent laryngeal nerve passes around what artery? A. Right auricle B. Right subclavian artery C. Right brachiocephalic artery D. Arch of the aorta E. Left auricle

D. Lateral femoral cutaneous nerve

The sensory nerve for lateral thigh runs inferolateral on the thigh and is deep to the inguinal ligament A. Vagus nerve B. Genitofemoral nerve C. Ilioinguinal nerve D. Lateral femoral cutaneous nerve

Lateral thoracic artery The lateral thoracic artery is a branch off the axillary artery

The serratus anterior muscle receives it blood supply from what artery?

Long thoracic nerve

The serratus anterior muscle receives it innervation from what nerve

A. Is attached to the endometrial epithelium verify

The seven-day blastocyst A. Is attached to the endometrial epithelium B. Has a single layer of trophoblast at the embryonic pole C. Has an amniotic cavity D. Is surrounded by a zona pellucida E. Is called the hypoblast

T10

The skin of the umbilicus receives its sensory innervation from:

E. none of the above

The space between the split layers of mesoderm is the: A. amniotic cavity B. blastocele C. foregut D. hindgut E. none of the above

b. tracheobronchial lymph nodes

The superficial and deep lymphatic drainage of the lungs unites at the hilus. Which group of lymph nodes do they drain into? a. thoracic duct b. tracheobronchial c. bronchomediastinal d. parasternal e. pulmonary

External oblique aponeurosis

The superficial inguinal ring is an opening in?

Internal thoracic vein

The superior epigastric vein drains into what?

E. Sternal angle and T4

The superior mediastinum is delimited from the inferior mediastinum by a plane crossing the: A. Ascending aorta and T3 B. Brachiocephalic trunk and descending aorta C. Sternoxyphoid joint and T4 D. Pulmonary valve and aortic arch E. Sternal angle and T4

c. manubrium

The superior mediastinum is mostly situated posterior to which of the following? a. arch of the aorta b. heart c. manubrium d. first rib e. body of the sternum

d. splenic artery

The suprarenal gland is supplied by branches of all EXCEPT which of the following? a. inferior phrenic artery b. renal artery c. abdominal aorta d. splenic artery

True Neural crest cells give rise to dorsal root ganglia and neurons, the peripheral ganglia of the sympathetic & parasympathetic nervous systems, odontoblasts, meninges, mesenchyme of the head & neck, Schwann cells, adrenal medulla and pigment cells of the skin.

The sympathetic nervous system is derived mostly from neural crest cells. true or false

D. Atrial septal defect

The tetralogy of Fallot includes all the following, EXCEPT: A. Pulmonary stenosis B. Overriding of the aorta C. Right ventricular hypertrophy D. Atrial septal defect E. Ventricular septal defect

A. Aortic hiatus

The thoracic duct enters the thoracic cavity through which diaphragm aperture? A. Aortic hiatus B. Inferior vena cava C. Esophageal hiatus

d. has C-shaped rings of cartilage supporting its lumen

The trachea: a .bifurcates in the anterior mediastinum b. lies anterior to the left brachiocephalic vein c. divides into two mainstem bronchi which are equal in diameter d. has C-shaped rings of cartilage supporting its lumen e. is lined by a stratified squamous epithelium

Intercostal nerves

The transversus thoracic muscle recieves innervation from what nerve?

A. Incorporation of the lower end of the mesonephric ducts

The trigone on the posterior wall of the urinary bladder is formed by the A. Incorporation of the lower end of the mesonephric ducts B. Incorporation of the lower end of the pronephric ducts C. Incorporation of the metanephric blastema D. Incorporation of the mesonephric tubules E. Incorporation of the pronephric tubules

B. Psoas major

The ureter lies against the anterior surface of which of the following muscles? A. Transversus abdominis B. Psoas major C. Quadratus lumborum D. Iliacus E. Crus of the diaphragm

a. Mesonephric duct

The ureteric bud appears as a dorsal outgrowth from the: a. Mesonephric duct b. Intermediate mesoderm c. Urogenital sinus d. Metanephric mesoderm e. Cloaca

A. Right lower quadrant

The usual location for an appendectomy incision is the A. Right lower quadrant B. Left lower quadrant C. Left upper quadrant D. Right upper quadrant

C. is the site of primordial germ cell production

The yolk sac in the human embryo: A. does not contribute to the embryonic gut B. is devoid of hemopoietic activity, or blood cell formation C. is the site of primordial germ cell production D. stores nutrients throughout pregnancy

B. right phrenic

This nerve lies on the lateral side of the superior vena cava before passing anterior to the root of the lung: A. right vagus B. right phrenic C. left phrenic D. left vagus

D. Psoas fascia

This structure covers the posterior wall between the parietal peritoneum and the muscles, continuous with the transversalis fascia, and is named according to the structure that it covers. A. Psoas major B. Psoas minor C. All of the above D. Psoas fascia

A. vitelline duct aka omphalomesenteric duct

This structure is a connection between embryonic midgut and yolk sac, from which gut developed. As development proceeds, the structure normally regresses. If it does not, it may persist as a Meckel's diverticulum, a cyst, a fistula, or a fibrous cord connecting gut to umbilicus. Meckel's diverticulum is an adult remnant of the: A. vitelline duct B. dorsal pancreatic duct C. hindgut D. pars cystica E. urachus

D. Ileocolic artery

Two days after an appendectomy on a 45-year old male patient, the patient has developed an elevated temperature (39 degrees celcius), is hypotensive, and complains of abdominal pain. An exploratory laparotomy reveals large amounts of blood in the pertioneal cavity due to an injury to a vessel that occured during the appendectomy. Which of the following vessels must be ligated to stop the bleeding? A. Superior mesenteric artery B. Right colic artery C. Right colic artery ad superior rectal artery D. Ileocolic artery E. Ileocolic artery and middle colic artery

Ostium primium and Ostium secundum

Typically, the abnormal flow of blood in a congenital atrial septal defect (ASD) results from overlapping of the:

a. Degeneration of the Pronephros

Unilateral renal agenesis was detected by ultrasonography in a near-term pregnancy that was associated with oligohydramnios. Failure of the kidney to develop probably resulted from: a. Degeneration of the Pronephros b. Degeneration of the mesonephros c. Ureteric duplication d. Failure of a ureteric bud to form e. Development of the paraxial mesoderm

A. Sagittal (Verify)

What anatomical plane (section) is represented in the images? A. Sagittal B. Transverse C. Oblique D. Frontal E. Coronal

Anterior abdominal wall and superior rectus abdominis muscles

What area does the superior epigastric vein drain?

The right anterior cardinal vein

What becomes the superior vena cava?

Right and left umbilical veins

What embryonic vessels gives rise to the ductus venosus?

Umbilical vein

What fetal blood vessel becomes the round ligament of the liver (ligamentum teres hepaticus)?

Protraction and stabilization of the scapula, as well as assisting in upward rotation

What is the function of the serratus anterior muscle?

To depress the ribs during forced expiration, to decrease the volume of the thoracic cavity

What is the function of the transversus thoracis?

D. Prevents polyspermy verify

What is the purpose of cortical granules in the cytoplasm of the egg? A. Is essential for fertilization B. Is caused by the zona pellucida C. Occurs in the male reproductive tract D. Prevents polyspermy E. Removes the head of the sperm

B. It allows for differentiation The primitive streak is a linear band of thickened epiblast that first appears at the caudal end of embryo and grows cranially. It proliferates to form the primitive knot or node. It is then possible to distinguish cranial (knot) and caudal (streak) ends of embryo. It does not specifically extend to form oropharyngeal membrane and cloacal membrane. Mesenchymal cells migrate from primitive knot to form notochordal process which then extend to form oropharyngeal membrane and cloacal membrane.

What is the significance of primitive streak? A. It allows for the differentiation between the three layers of trilaminar disc. B. It allows for differentiation between caudal and cranial ends of epiblast. C. It specifically extends cranially and caudally to form oropharyngeal membrane and cloacal membrane. D. It forms the 4 limbs of the body. E. B and C

D. Choriocarcinoma Choriocarcinoma is a malignant tumor that arises from trophoblastic cells that may occur following normal pregnancy, abortions, or hydatidiform mole.

What malignant tumor arises from trophoblastic cells? A. Liposarcoma B. Rhabdomyosarcoma C. Fibroma D. Choriocarcinoma E. Endometrioma

Serratus anterior muscle

What muscle is innervated by the long thoracic nerve?

Rectus abdominis

What muscle originates at the pubic crest and is innervated by thoracic spinal nerves T7-T11 AND T12?

Left ventricle

What part of the heart forms the left border?

Right atrium

What part of the heart forms the right border?

E. Inner cell mass

What portion of the blastocyst will give rise to the embryo? A. Outer cell mass B. Ectoderm C. Cytotrophoblast D. Endoderm E. Inner cell mass

C. gastrulation Gastrulation is the process that converts epiblast from a bilaminar disc into a trilaminar embryonic disc, consisting of the three main layers: ectoderm, mesoderm, and endoderm.

What process converts the bilaminar disc into the trilaminar embryonic disc consisting of ectoderm, mesoderm, and endoderm? A. somitogenesis B. transformation C. gastrulation D. neurulation

C. Tail of pancreas

What structure lies near the hilum of the spleen and upper plate of left kidney? A. Head of the pancreas B. Body of the pancreas C. Tail of pancreas D. Tail and head of pancreas

A. Vitelline duct verify

What structure provides a connection between the embryonic midgut and yolk sac, from which the gut develops. A. Vitelline duct B. Dorsal pancreatic duct C. Hindgut D. Pars cystica E. Urachus

Inferior epigastric artery and vein, pubic artery and vein and the superficial epigastric vessels Cremasteric artery and vein and testicular artery and vein could be affected as well (if the patient is male)

What vasculature structures might be cut during a Pfannenstiel incision?

Hemiazygos vein

What vein drains the lower third of the thoracic wall?

c. right gastric and right gastro-omental arteries

When performing a surgical repair of a bleeding ulcer in the pyloric antrum, it is important to know that the principal arterial supply to the pyloric antrum is the: a. gastroduodenal and short gastric arteries b. left gastric and short gastric arteries c. right gastric and right gastro-omental arteries d. gastroduodenal artery e. left gastric and right gastric arteries

c. dorsal root ganglia at T1-T4

Where are the cell bodies of afferent fibers which conduct pain sensation from the heart located? a. inferior vagal ganglion b. intermediolateral horn at T1-T4 c. dorsal root ganglia at T1-T4 d. superior, middle, and inferior cervical ganglia e. paravertebral ganglia at T1-T4

A. Posterior

Where is the pancreas is relation to the stomach? A. Posterior B. Anterior C. Superior D. None of these

B. Anterior and Superior

Which best describes the location of the common bile duct in relation to the pancreatic head? A. Posterior and Inferior B. Anterior and Superior C. Posterior and Lateral D. Posterior and Superior E. Anterior and Lateral

D. Ectoderm, mesoderm, and endoderm During week 3 of development, the process of gastrulation, which establishes the three primary germ layers (ectoderm, intraembryonic mesoderm, and endoderm), occurs. The origin of all tissues and organs of the adult can be traced to one of these germ layers because these are whence they "germinate."

Which germ layers are present at the end of week 3 of development (day 21)? A. Epiblast only B. Epiblast and hypoblast C. Ectoderm and endoderm D. Ectoderm, mesoderm, and endoderm E. Epiblast, mesoderm, and hypoblast

D. Extends the torso

Which is not true of the external oblique muscles? A. Nerve supply T7-T11 B. Forms the inguinal ligament C. Originates at the external aspects of lower 8 ribs D. Extends the torso

C. syncytiotrophoblast

Which layer invades into the maternal endometrium and forms the lacunar network for placental-maternal exchange? A. blastocyst B. cytotrophoblast C. syncytiotrophoblast D. endoderm E. ectoderm F. mesoderm

A. syncytiotrophoblast

Which layer is acellular, does not expand mitotically, and produces human chorionic gonadotropin (hCG) which stimulates progesterone production by the corpus luteum? A. syncytiotrophoblast B. cytotrophoblast C. epiblast D. endoderm E. ectoderm F. mesoderm

B. cytotrophoblast

Which layer is cellular and expands mitotically into syncytiotrophoblast to form primary chorionic villi that can be used in chorionic villus sampling or genetic testing for the early fetus? A. syncytiotrophoblast B. cytotrophoblast C. epiblast D. endoderm E. ectoderm F. mesoderm

A. epiblast The bilaminar embryonic disc comprises the epiblast and the hypoblast. Epiblast forms part of the lining of the amniotic cavity, while the hypoblast lines part of the primitive yolk sac.

Which layer of the bilaminar embryonic disc forms part of the lining of the amniotic cavity? A. epiblast B. hypoblast C. blastocyst D. trophoblast E. syncytiotrophoblast F. cytotrophoblast G. blastocoele H. inner cell mass I. notochord J. prechordal plate K. primary yolk sac L. amniotic cavity M. intraembryonic coelom N. secondary yolk sac

E. hand plates

Which of the following are NOT distinctive characteristics of the fourth week of development? A. neuropores B. somites C. branchial arches D. lower limb buds E. hand plates

b. it drains the left gonadal vein

Which of the following best describes the left renal vein? a. it is shorter than the right renal vein b. it drains the left gonadal vein c. it lies posterior to the renal pelvis d. it is part of the hepatic portal system e. it terminates in the cysterna chili

A. the anterior cardiac veins open directly into the right atrium

Which of the following descriptions of the blood vessels serving the heart is TRUE? A. the anterior cardiac veins open directly into the right atrium B. the great cardiac vein accompanies the right coronary artery C. the posterior interventricular artery accompanies the small cardiac vein D. the artery to the atrioventricular node is a branch of the anterior interventricular artery E. the apex of the heart is supplied by the right marginal artery

F. all of the above arise from neural crest

Which of the following does NOT arise from the neural crest? A. spinal ganglia B. pigment cells C. suprarenal medulla D. skeletal and muscular components in the head E. sheaths of peripheral nerves F. all of the above arise from neural crest

D. Pleuropericardial membranes

Which of the following does not contribute to the diaphragm? A. Body wall musculature B. Pleuroperitoneal membranes C. Septum transversum D. Pleuropericardial membranes E. Esophageal mesentery

A. herniation of midgut loop

Which of the following events in the development of the abdominal cavity are greatly affected by the rapid growth of the liver: A. herniation of midgut loop B. urorectal septum formation C. degeneration of the ventral mesentery D. formation of inferior recess of lesser sac E. dorsal mesentery morphogenesis

A. Sigmoid colon

Which of the following is NOT supplied by the branches of the superior mesenteric artery? A. Sigmoid colon B. Duodenum C. Pancreas D. Appendix E. Transverse colon

A. transverse mesocolon

Which of the following is a derivative of the embryonic dorsal mesentery? A. transverse mesocolon B. hepatoduodenal ligament C. lesser omentum D. falciform ligament E. hepatogastric ligament

B. Ductus arteriosus

Which of the following is a fetal shunt that diverts blood from pulmonary artery directly into the aorta: A. Ductus venosus B. Ductus arteriosus C. Ligamentum arteriosus D. Foramen ovale E. Umbilical artery

A. omphalocele

Which of the following is an abdominal herniation covered by amnion? A. omphalocele B. volvulus C. Meckel's deverticulum D. diaphragmatic hernia E. gastroschisis

C. epithelial lining of the respiratory tract The endoderm gives rise to the epithelial lining and associated glands of the GI, respiratory, and lower urogenital tracts.

Which of the following is derived from ENDODERM? A. endoneurial fibroblasts and Schwann cells of peripheral nerves B. endothelial lining of blood vessels C. epithelial lining of the respiratory tract D. cells lining the amniotic membrane E. cytotrophoblast placental cells F. NONE of the above

E. ureter Kidneys and ureter develop from the intermediate mesoderm. Somites and their derivatives develop from paraxial mesoderm, and lateral plate mesoderm becomes either connective tissues of the body wall and limbs (somatic layer of lateral plate mesoderm) or gut wall (splanchnic layer).

Which of the following is derived from the intermediate mesoderm? A. enamel of the teeth B. smooth muscle of the gut C. mucous lining of the larynx D. sympathetic chain ganglia E. ureter F. suprarenal medulla G. patella H. skeletal muscle of the upper limb I. heart J. lining of the lungs K. spinal cord L. parenchyma of the pancreas M. eleventh thoracic vertebra N. aortic arch

B. smooth muscle of the gut The lining of the gut is endodermal in origin, but the smooth muscle of the gut forms from the splanchnic layer of the lateral plate mesoderm. Visceral peritoneum also forms from splanchnic mesoderm, but the parietal layers of the serous membranes (peritoneum, pleura, pericardium) form from the somatic layer of lateral plate mesoderm.

Which of the following is derived from the splanchnic layer of the lateral plate mesoderm? A. enamel of the teeth B. smooth muscle of the gut C. mucous lining of the larynx D. sympathetic chain ganglia E. ureter F. suprarenal medulla G. patella H. skeletal muscle of the upper limb I. heart J. lining of the lungs K. spinal cord L. parenchyma of the pancreas M. eleventh thoracic vertebra N. aortic arch

A. umbilical vein

Which of the following is most closely associated with the ligamentum teres hepatis is: A. umbilical vein B. 6th aortic arch C. vitelline vein D. 3rd aortic arch E. umbilical artery

A. Umbilical vein

Which of the following is most closely associated with the ligamentum teres hepatis is: A. Umbilical vein B. 6th aortic arch C. Vitelline vein D. 3rd aortic arch E. Umbilical artery

A. ventricular septal defect

Which of the following is the most common congenital cardiac malformation? A. ventricular septal defect B. atrial septal defect C. dextrocardia D. tetralogy of Fallot E. transposition of great vessels

b. it receives venous blood from most of the gastrointestinal tract

Which of the following is true of the hepatic portal vein? a. it is formed by the union of the superior mesenteric and inferior mesenteric veins b. it receives venous blood from most of the gastrointestinal tract c. it passes to the liver through the falciform ligament d. it forms anterior to the head of the pancreas

a. transverse colon

Which of the following organs is suspended on a mesentery in the adult? a. transverse colon b. descending colon c. ascending colon d. second part of the duodenum e. pancreas

D. 26 weeks - 1000 gm.

Which of the following statements about fetal age and weight shows a normal relationship? A. 8 weeks - 10 gm. B. 12 weeks - 200 gm. C. 20 weeks - 800 gm. D. 26 weeks - 1000 gm. E. 38 weeks - 4600 gm.

C. notochord The buccopharyngeal membrane, heart, pericardial cavity and septum transversum all end up as ventral structures in the embryo, whereas the notochord remains dorsal. Remember, though, that before the folding of the embryo, the heart begins its development extraembryonically in a region anterior to the prochordal plate.

Which of the following structures does not turn under onto the ventral surface of the embryo during folding of the head? A. prechordal plate B. heart C. notochord D. pericardial cavity E. septum transversum

D. epidermis Mesenchyme is a term used interchangeably with mesoderm, which forms many structures, including muscle, cartilage, blood cells and blood vessels. Epidermis, on the other hand, is of ectodermal origin, not mesenchyme. Realize, though, that the dermis, just below the epidermis, is of mesenchymal origin.

Which of the following structures is NOT derived from mesenchyme? A. muscle fiber B. cartilage of the limbs C. white blood cell precursor D. epidermis E. blood vessel

B. notochord The notochord is thought to be an important structure in induction of nervous system development, axial skeleton development and other organogenic events.

Which of the following structures is believed to be a primary organizer or inducer during organogenesis? A. Somites B. notochord C. metanephric blastema D. lens placode E. none of the above

H. skeletal muscle of the upper limb Paraxial mesoderm forms the somites, which give rise to the axial skeleton and connective tissues of the trunk, all skeletal muscle, and the dermis. In the lower cervical regions, the somitic myotomes form the skeletal muscle of the developing upper limb. The eleventh thoracic vertebra is sclerotomal (somitic), but from lower thoracic levels.

Which of the following structures is derived from paraxial mesoderm of the cervical region? A. enamel of the teeth B. smooth muscle of the gut C. mucous lining of the larynx D. sympathetic chain ganglia E. ureter F. suprarenal medulla G. patella H. skeletal muscle of the upper limb I. heart J. lining of the lungs K. spinal cord L. parenchyma of the pancreas M. eleventh thoracic vertebra N. aortic arch

B. Zona pellucida zona pellucida degenerates and is replaced by the underlying cells called the trophoblast

Which of the following structures must degenerate for blastocyst implantation to occur? A. Endometrium in progestational phase B. Zona pellucida C. Syncytiotrophoblast D. Cytotrophoblast E. Functional layer of the endometrium

External oblique muscle, skin, peritoneum

Which of these are layers of the Anterior Abdominal Wall?

A. Right upper

Which one of the following is not one of the nine regions of the abdomen? A. Right upper B. Right hypochondriac C. Left inguinal or iliac D. Epigastric E. Left lumbar

(B) Gastrulation Gastrulation establishes the three primary germ layers during week 3 of development. Neurulation is the process by which neuroectoderm forms the neural plate, which eventually folds to form the neural tube.

Which process establishes the three definitive germ layers? (A) Neurulation (B) Gastrulation (C) Craniocaudal folding (D) Lateral folding (E) Angiogenesis

E. none of the above

Which statement about the 14-day blastocyst is NOT true? A. villi are absent B. extraembryonic coelom surrounds the yolk sac C. primitive uteroplacental circulation is established D. extraembryonic mesoderm is split into two layers E. none of the above

D. Linea alba

Which structure is devoid of nerves and blood vessels and is often the site of midline surgical entry into the abdomen? A. Costal cartilages of last 3 ribs B. Pubic crest C. Pyramidalis D. Linea alba

D. Posterior wall of stomach

Which structure is most common mistaken for the pancreas A. Parts of left lobe of liver B. Lymph nodes C. Caudate surface of liver D. Posterior wall of stomach E. Horseshoe kidney

B. Fewer mesenteric arterial arcades The jejunum has fewer mesenteric arterial arcades but longer vasa recta than the ileum. The plicae circulares (circular folds) are tall and closely packed in the jejunum and are low and sparse in the ileum, and the lower part of the ileum has no plicae circulares. More digestion and absorption of nutrients occurs in the jejunum than in the ileum, and less fat is found in the mesentery of the jejunum.

While examining radiographs and angiograms of a 52-year-old patient, a physician is trying to distinguish the jejunum from the ileum. He has observed that the jejunum has: A. Fewer plicae circulares B. Fewer mesenteric arterial arcades C. Less digestion and absorption of nutrients D. Shorter vasa recta E. More fat in its mesentery

D. sensory nuclei in the hindbrain lie ventral to motor nuclei

With respect to the nervous system, which of the following is NOT correct: A. a muscle's nerve supply can be used as an indicator of its level of origin and path of migration B. Rathke's pouch is an outgrowth of oral ectoderm that becomes the anterior lobe of the hypophysis C. the sulcus limitans marks the boundary between motor and sensory areas D. sensory nuclei in the hindbrain lie ventral to motor nuclei

A. SMA and SMV

You are at surgery and are about to mobilize the second portion of the duodenum and the head of the pancreas. You note an artery and vein passing anterior to the uncinate process of the pancreas and the third portion of the duodenum. These vessels are the: A. SMA and SMV B. inferior mesenteric artery and vein C. gastroduodenal artery and vein D. superior pancreaticoduodenal artery and vein E. middle colic artery and vein

e. ninth intercostal space

You must remove fluid from the pleural cavity of your patient (thoracocentesis). You decide to insert the aspiration needle over the top of a rib, into an intercostal space inferior to the lower border of the lung in the MAL (mid axillary line) at the end of a normal expiration. At which level might this procedure be safely done without injuring the lung? a. fourth intercostal space b. fifth intercostal space c. sixth intercostal space d. seventh intercostal space e. ninth intercostal space

b. paraumbilical vein and small epigastric vein

Your patient who has cirrhosis and has developed varices "caput medusae". This is due to dilatation of the anastomosis between which of the following pairs of veins? a. left gastric vein and the azygos b. paraumbilical vein and small epigastric vein c. portal vein and retroperitoneal veins d. portal vein and inferior vena cava

A. Greater curvature of the stomach

a 29 year old male was taken to the emergnecy room after a car accident. Examination showed low blood pressure, cold damp skin and tenderness on the left mid axilarry line X-rays revealed that his 9th-10th ribs were fractured near their angles. During the splenectomy, the surgeon ligates the blood supply of the spleen at its hilum. Improper placement of the ligature may damage the arterial supply to which of the following structures? A. Greater curvature of the stomach B. Pyloric portion of the stomach C. Splenic flexure of the colon D. Tail of the pancreas E. Lesser curvature of the stomach


Set pelajaran terkait

Chapter 6 Onboarding and Training

View Set

A History of Western Society chapter 17

View Set

Psychology of Learning Exam 3 (Chp. 5) Prep Guides 7&8

View Set

Ch. 51: Assessment and Management of Patients With Diabetes

View Set

Classroom Assessment Final Exam 1-16

View Set

Object Oriented Programming Interview Questions

View Set

Ch 40: Management of Patients with Gastric and Duodenal Disorders

View Set